<<

Website: www.EminentIAS.com Telegram: @eminentias_info

QUESTION: With reference to the Defence Acquisition Procedure 2020, consider the following statements:

1. The government has decided to remove the clause for offsets if the equipment is being bought either through deals or agreements between two countries, or through an ab initio single-vendor deal.

2. The offset is an obligation by an international player to boost India’s domestic defence industry if India is buying defence equipment from it.

Which of the statements given above is/are correct?

A. 1 only

B. 2 only

C. Both 1 and 2

D. Neither 1 nor 2

Answer: C

Explanation :

The Defence Ministry came up with its latest Defence Acquisition Procedure 2020 (DAP 2020), which comes into effect from Thursday, October 1.

 Changing a 15-year old policy, the government has decided to remove the clause for offsets if the equipment is being bought either through deals or agreements between two countries, or through an ab initio single-vendor deal.

 The offset is an obligation by an international player to boost India’s domestic defence industry if India is buying defence equipment from it.

 Since defence contracts are costly, the government wants part of that money either to benefit the Indian industry, or to allow the country to gain in terms of technology.

 Offsets is a “mechanism generally established with the triple objectives of:

o partially compensating for a significant outflow of a buyer country’s resources in a large purchase of foreign goods,

o facilitating induction of technology and

o adding capacities and capabilities of domestic industry”.  An offset provision in a contract makes it obligatory on the supplier to either “reverse purchase, execute export orders or invest in local industry or in research and development” in the buyer’s domestic industry, according to CAG.

 The policy was adopted on the recommendations of the Vijay Kelkar Committee in 2005. The first offset contract was signed in 2007.

 Hence both statements are correct.

QUESTION: With reference to the Supersonic Missile Assisted Release of Torpedo (SMART), consider the following statements: 1. SMART is a missile assisted release of lightweight Anti-Submarine Torpedo System for Anti- Submarine Warfare (ASW) operations far beyond Torpedo range. 2. It has been successfully flight tested from Wheeler Island off the coast of . Which of the statements given above is/are correct? A. 1 only B. 2 only C. Both 1 and 2 D. Neither 1 nor 2 Answer: C Explanation : Supersonic Missile Assisted Release of Torpedo (SMART) has been successfully flight tested from Wheeler Island off the coast of Odisha. • SMART is a missile assisted release of lightweight Anti-Submarine Torpedo System for Anti-Submarine Warfare (ASW) operations far beyond Torpedo range. • This launch and demonstration is significant in establishing Anti-Submarine warfare capabilities. • A number of DRDO laboratories including DRDL, RCI Hyderabad, ADRDE Agra, NSTL Visakhapatnam have developed the technologies required for SMART. • Hence both statements are correct.

News: https://timesofindia.indiatimes.com/india/drdo-successfully-flight-tests-weapon-system- smart/articleshow/78489306.cms

QUESTION: With reference to a report “Economic valuation of ecosystem services, National Zoological Park, New Delhi’, consider the following statements: 1. It highlights the importance of habitats such as zoos to human wellbeing and the need of replication across India.

2. It was released by Centre for Science and Environment.

Which of the statements given above is/are correct?

A. 1 only

B. 2 only

C. Both 1 and 2 D. Neither 1 nor 2

Answer: A

Explanation :

Union Minister of Environment launched a report of the CZA-TERI titled “Economic valuation of ecosystem services, National Zoological Park, New Delhi ‘.

 The report by Central Zoo Authority (CZA) and TERI highlights the importance of habitats such as zoos to human wellbeing and the need of replication across India.

 The study first of its kind in India and perhaps the entire World, pegs the total annual economic value of the ecosystem services (biodiversity conservation, employment generation, carbon sequestration, education and research, recreational and cultural) at around 423 crore (2019-20).

 The total value of the one-time cost of services such as carbon storage and land value provided by the zoo is estimated to be around 55,000 crore.

Hence, option (a) is the correct answer.

News: https://pib.gov.in/PressReleasePage.aspx?PRID=1661727

QUESTION: With reference to the emission norms for tractors, consider the following statements: 1. It will be applicable from October 2021 and for Construction Equipment Vehicles, the norms will be applicable from April 2021.

2. The amendment to Central Motor Vehicle Rules 1989 includes separate emission norms for agricultural machinery (agricultural tractors, power tillers and combined harvesters) and construction equipment vehicles.

Which of the statements given above is/are correct?

A. 1 only

B. 2 only

C. Both 1 and 2

D. Neither 1 nor 2

Answer: C

Explanation :

Ministry of Road Transport and Highways has said that emission norms for tractors will be applicable from October 2021 and for Construction Equipment Vehicles, the norms will be applicable from April 2021.  The Ministry has notified amendment to Central Motor Vehicle Rules 1989 on 30th September, 2020 deferring the applicability date for implementing the next stage of emission norms for tractors and Construction Equipment Vehicles.

 The amendment includes separate emission norms for agricultural machinery (agricultural tractors, power tillers and combined harvesters) and construction equipment vehicles.

 There is also change in the nomenclature of emission norms from Bharat Stage (CEV/TREM)IV and Bharat Stage (CEV/TREM)V to TREM Stage-IV and TREM Stage-V for agricultural tractors and other equipment.

 The Ministry had received the request in this regard from the Ministry of Agriculture, Tractor Manufacturers and agriculture associations.

 Hence both statements are correct.

News: https://www.business-standard.com/article/economy-policy/govt-extends-deadline-for-new- emission-norms-for-tractors-to-oct-2021-120100501271_1.html

QUESTION: With reference to education for deaf children, consider the following statements: 1. Recently a MoU signed between Indian Sign Language Research and Training Center (ISLRTC) and NCERT to make education materials accessible for Deaf children in their preferred format of communication.

2. The Indian Sign Language Research and Training Center (ISLRTC), Delhi is a Society under the Department of Empowerment of Persons with Disabilities, the Ministry of Social Justice and Empowerment.

Which of the statements given above is/are correct?

A. 1 only

B. 2 only

C. Both 1 and 2

D. Neither 1 nor 2

Answer: C

Explanation :

A historic MoU will be signed on October 6, 2020 between Indian Sign Language Research and Training Center (ISLRTC) and NCERT to make education materials accessible for Deaf children in their preferred format of communication.

 As part of this MoU, educational print materials including NCERT Textbooks, Teachers Handbook and other Supplementary materials and resources of Class I-XII of all subjects of both Hindi and English Medium will be converted into Indian Sign Language in Digital format.  The signing of this MoU is seen as a historic step as availability of NCERT textbooks in Indian Sign Language will ensure that Deaf children can also now access educational resources in Indian Sign Language.

 This will also ensure educational standardization of Indian Sign Language as mandated in the New Education Policy, 2020.

 The Indian Sign Language Research and Training Center (ISLRTC), Delhi is a Society under the Department of Empowerment of Persons with Disabilities, the Ministry of Social Justice and Empowerment.

 Hence both statements are correct.

Source: http://newsonair.com/News?title=ISLRTC-%26-NCERT-to-ink-MoU-to-make-education- materials-accessible-for-Deaf-children&id=401469

QUESTION: Consider the following statements regarding Healthy and Energy Efficient Buildings Initiative: 1. It is launched by Energy Efficiency Services Limited (EESL). 2. It is supported by the World Bank. 3. It will address the challenges of retrofitting existing buildings and air conditioning systems. Select the correct codes given below: (a) 1 only (b) 2 and 3 only (c) 1 and 3 only (d) 3 only Ans: (c) 1 and 3 only It is launched by Energy Efficiency Services Limited (EESL), in partnership with the U.S. Agency for International Development’s (USAID) MAITREE program. This initiative will address the challenges of retrofitting existing buildings and air conditioning systems so that they are both healthy and energy-efficient. #News: It is a recent initiative.

QUESTION: Consider the following statements regarding Mission Purvodaya: 1. It aims to develop the eastern region of India into an integrated steel hub. 2. It will be implemented with the assistance of Singapore. Select the correct codes given below: (a) 1 only (b) 2 only (c) Both 1 and 2 (d) Neither 1 nor 2 Ans: (a) 1 only The Centre unveils Mission Purvodaya to develop eastern region into an integrated steel hub. The eastern belt has the potential to add over 75 percent of the country’s incremental steel capacity envisioned by the National Steel Policy. Through this programme, the government aims to transform logistics and utilities infrastructure which would change the socio-economic landscape in the eastern India. The steps, under mission, also include growth of steel industry along with employment opportunities across the entire value chain. It will be implemented with the assistance of Japan. #News: It is a recent initiative.

QUESTION: With reference to the Stockholm Convention on Persistent Organic Pollutants (POPs), Consider the following statements: 1. India has ratified the Stockholm Convention but new amendments in various Annexes of the convention cannot be enforced on it unless further ratified by India. 2. Ministry of Chemicals and Fertilizers has powers to ratify chemicals under the Convention in respect of domestically regulated POPs. Select the correct codes given below: a) 1 only b) 2 only c) Both 1 and 2 d) Neither 1 nor 2 Correct answer: A Explanation: Statement 1 is correct. India had ratified the Stockholm Convention on January 13, 2006 as per Article 25(4), which enabled it to keep itself in a default "opt-out"position such that amendments in various Annexes of the convention cannot be enforced on it unless an instrument of ratification/ acceptance/ approval or accession is explicitly deposited with UN depositary. Statement 2 is incorrect. The Union Cabinet recently delegated its powers to ratify chemicals under the Stockholm Convention to Union Ministers of External Affairs (MEA) and Environment, Forest and Climate Change (MEFCC) in respect of POPs already regulated under the domestic regulations thereby streamlining the procedure.

QUESTION: With reference to the Gyan Circle Ventures, consider the following statements: 1. It is a Technology Business Incubator (TBI) funded by the Ministry of Electronics and Information Technology. 2. It aims to provide investments, infrastructure and mentoring support for innovation and startups. Select the correct codes given below: a) 1 only b) 2 only c) Both 1 and 2 d) Neither 1 nor 2 Correct answer: C Explanation: Statement 1 is correct. The Minister of Education recently inaugurated the Gyan Circle Ventures, a Ministry of Electronics and Information Technology funded Technology Business Incubator (TBI) of Indian Institute of Information Technology, Sri City (Chittoor), Andhra Pradesh. Gyan Circle Ventures would function as a Technology Incubation and Development of Entrepreneurs (TIDE 2.0) incubation center as approved by the Ministry of Electronics and Information Technology (MeitY). TIDE 2.0 scheme promotes tech entrepreneurship through financial and technical support to incubators engaged in supporting ICT startups primarily engaged in using emerging technologies such as IoT, AI, Block-chain, Robotics etc. in pre-identified areas of societal relevance. Statement 2 is correct. The Gyan Circle Ventures will serve as a hub for innovation and startups by providing support, in various phases, via investments, infrastructure and mentoring. The TBI would have an Advisory Committee comprising leading Industrialists, entrepreneurs and technical experts. It would enable incubatees to leverage these expert mentors and networks from both academia and the industry.

QUESTION: With reference to the Production Linked Incentive Scheme (PLI), Consider the following statements 1. It provides a financial incentive to boost domestic manufacturing and attract large investments in the electronics value chain in India. 2. The scheme covers all electronics items manufacturing in India except mobile phones. Select the correct codes given below: a) 1 only b) 2 only c) Both 1 and 2 d) Neither 1 nor 2 Correct answer: A Explanation: Statement 1 is correct. The Production Linked Incentive Scheme (PLI) for Large Scale Electronics Manufacturing proposes a financial incentive to boost domestic manufacturing and attract large investments in the electronics value chain including electronic components and semiconductor packaging. Incentive: The Scheme extends an incentive of 4% to 6% on incremental sales (over base year) of goods manufactured in India and covered under target segments, to eligible companies, for a period of five years subsequent to the base year 2019-20. Statement 2 is incorrect. The Scheme is only applicable for target segments namely mobile phones and specified electronic components. Specified Electronic Components: SMT components, Discrete semiconductor devices including transistors, diodes, thyristors, etc., Passive components including resistors, capacitors, etc. for electronic applications, Printed Circuit Boards (PCB), PCB laminates, prepregs, photopolymer films, PCB printing inks, Sensors, transducers, actuators, crystals for electronic applications, System in Package (SIP), Micro or Nano-electronic components such as Micro Electromechanical Systems (MEMS) and Nano Electromechanical Systems (NEMS), and Assembly, Testing, Marking and Packaging (ATMP) units.

QUESTION: With reference to the Food Loss Index (FLI), Consider the following statements : 1. It examines the food loss along supply activities such as production, handling & storage, and processing. 2. It has been developed by the International Food Policy Research Institute (IFPRI). Select the correct codes given below: a) 1 only b) 2 only c) Both 1 and 2 d) Neither 1 nor 2 Correct answer: A Explanation: Sustainable Development Goal 12: Ensure sustainable consumption and production patterns. SDG Target 12.3 seeks to halve global food waste at retail and consumer levels, as well as to reduce food loss during production and supply. In order to measure food waste and losses, two indices have been proposed: Food Waste Index (FWI) and Food Loss Index (FLI). Statement 1 is correct. The Food Loss Index examines the food loss along supply activities such as production, handling & storage, and processing. It focuses on food losses that occur from production up to (and not including) the retail level. It measures the changes in percentage losses for a basket of 10 main commodities by country in comparison with a base period. Statement 2 is incorrect. To monitor SDG Target 12.3, Food and Agriculture Organization has created the Food Loss Index (FLI). The focus of the indicator is on percentages of food removed from the supply chain. Further, the Food Waste Index is currently in development at UN Environment. The FWI will measure tons of wasted food per capita, considering a mixed stream of products from processing through to consumption.

QUESTION: With reference to the Quadrilateral Security Dialogue (Quad), consider the following statements : 1. It has been formed by a mutual-assistance treaty for strategic cooperation in the Indo-pacific. 2. India is the only BRICS member country that is also part of the Quad. Select the correct codes given below: a) 1 only b) 2 only c) Both 1 and 2 d) Neither 1 nor 2 Answer: B Statement 1 is incorrect. The Quadrilateral Security Dialogue or Quad is an informal strategic forum between the United States, Japan, Australia and India that is maintained by semi-regular summits and information exchanges among the member countries. Quad, being an informal forum, has no formal mutual-assistance treaty, secretariat, or even a website. Statement 2 is correct. BRICS members are Brazil, Russia, India, China, and South Africa; India is the only country in Quad that is a member of BRICS.

QUESTION: With reference to the AstroSat, Consider the following statements : 1. It is first dedicated Indian astronomy mission aimed at studying celestial sources in Xray, optical and UV spectral bands simultaneously. 2. AstroSat is orbiting Earth in a Geo-Synchronous Orbit. Select the correct codes given below: a) 1 only b) 2 only c) Both 1 and 2 d) Neither 1 nor 2 Answer: A Statement 1 is correct. AstroSat is the first dedicated Indian astronomy mission aimed at studying celestial sources in X-ray, optical and UV spectral bandssimultaneously. The payloads cover the energy bands of Ultraviolet (Near and Far), limited optical and X-ray regime. One of the unique features of AstroSat mission is that it enables the simultaneous multiwavelength observations of various astronomical objects with a single satellite. # AstroSat has detected extreme UV light from a galaxy called AUDFs01, 9.3 billion lightyears away from Earth in a discovery by a global team led by scientists of the Inter University Centre for Astronomy and Astrophysics (IUCAA). Statement 2 is incorrect. AstroSat is a multi-wavelength astronomy mission on an IRS-class satellite in a 650-km,near-earth equatorial orbit. It was launched by the Indian launch vehicle PSLV from Satish Dhawan Space Centre, Sriharikota in 2015. # Geosynchronous orbit is a high Earth orbit that allows satellites to match Earth's rotation. Located at 22,236 miles (35,786 kilometers) above Earth's equator, this position is a valuable spot for monitoring weather, communications and surveillance. QUESTION: The purpose of the MOSAiC Expedition is: a) Experimental study of Outer Space plantation b) Study of Arctic Climate c) Global Climate Change study expedition d) Survey the global governmental response to COVID-19 Answer: B The goal of the MOSAiC expedition is to take the closest look ever at the Arctic as the epicenter of global warming and to gain fundamental insights that are key to better understand global climate change. Hundreds of researchers from 20 countries are involved in this exceptional endeavour. The MOSAiC is spearheaded by Alfred Wegener Institute, Helmholtz Centre for Polar and Marine Research (AWI), Germany. The ship Polarstern recently docked at its home port of Bremerhaven nearly 13 months after it left Norway. QUESTION: With reference to the Strengthening Teaching-Learning and Results for States (STARS) project, Consider the following statements : 1. It is to be implemented as a Centrally Sponsored Scheme with financial support from the World Bank. 2. It includes a Contingency Emergency Response Component to be more responsive to any natural, man-made and health disasters. Select the correct codes given below: a) 1 only b) 2 only c) Both 1 and 2 d) Neither 1 nor 2 Correct answer: C Explanation: Statement 1 is correct. Cabinet has recently approved the implementation of the Strengthening Teaching-Learning and Results for States (STARS) project with a total project cost of Rs 5718 crore with the financial support of World Bank amounting to US $ 500 million. STARS project would be implemented as a new Centrally Sponsored Scheme under Department of School Education and Literacy, Ministry of Education (MOE). Statement 2 is correct. STARS project has separate National and State level implementation components. It further includes a Contingency Emergency Response Component (CERC) under the National Component which would enable it to be more responsive to any natural, man-made and health disasters. It will help the government respond to situations leading to loss of learning such as school closures/infrastructure damage, inadequate facilities and use technology for facilitating remote learning etc. QUESTION: With reference to the SCALE platform launched by Leather Sector Skill Council (LSSC), consider the following statements : 1. It will help candidates to get assessed and certified on their knowledge and skills on a specific qualification. 2. The platform will serve as a job portal with access to a certified talent pool of candidates to hire from. Select the correct codes given below: a) 1 only b) 2 only c) Both 1 and 2 d) Neither 1 nor 2 Correct answer: C Explanation: Both statements are correct. The Leather Sector Skill Council (LSSC) recently announced the launch of Skill Certification Assessment for Leather Employees (SCALE) India web platform and android app. SCALE will help candidates in the skilling ecosystem to get assessed and certified on their knowledge, skills and behaviour on a specific qualification pack. It also enables access to micro-learning modules for bridging the skill gaps that are identified and can be re-assessed and certified at the end of the learning on the specific qualification pack. From an employer standpoint, the platform will serve as a job portal with access to a certified talent pool of candidates to hire from, leading to increase in productivity and profitability. Employers can share their skill gap requirements and check for suitable training modules available to bridge the skill gap of their workforce and raise training requests for short term/customised training that would be serviced by the LSSC. # LSSC was set up as a non-profit organisation in 2012 as one of the key sector skill councils approved by National Skill Development Corporation (NSDC). QUESTION: To which of the following issue The Dumbarton Oaks Conference is related: a) First ever international conference on Climate Change b) Proposals for a world organization that became the basis for the United Nations c) Israel—Palestine boundary negotiations d) Peaceful utilization of outer space entities Correct answer: B Explanation: Dumbarton Oaks Conference, 1944 in Washington, D.C., where representatives of China, the Soviet Union, the United States, and the United Kingdom formulated proposals for a world organization that became the basis for the United Nations. The Dumbarton Oaks proposals did not furnish a complete blueprint for the United Nations. They failed to provide an agreed arrangement on the voting system of the proposed Security Council and the membership provisions for the constituent republics of the Soviet Union. These issues were resolved at the Yalta Conference in February 1945. The proposals, as thus supplemented, formed the basis of negotiations at the San Francisco Conference, out of which came the Charter of the United Nations in 1945. QUESTION: With reference to the Electronic Vaccine Intelligence Network (eVIN), consider the following statements : 1. It is being implemented under National Health Mission by Ministry of Health and Family Welfare. 2. It aims to provide real-time information on vaccine stocks and flows, and storage temperatures across all cold chain points in the country. Select the correct codes given below: a) 1 only b) 2 only c) Both 1 and 2 d) Neither 1 nor 2 Correct answer: C Explanation: Statement 1 is correct. The Electronic Vaccine Intelligence Network (eVIN) is an innovative technological solution aimed at strengthening immunization supply chain systems across the country. This is being implemented under National Health Mission (NHM) by Ministry of Health and Family Welfare. Statement 2 is correct. eVIN aims to provide real-time information on vaccine stocks and flows, and storage temperatures across all cold chain points in the country. The Electronic Vaccine Intelligence Network (eVIN) system is being enhanced to address the needs for distribution and tracking of COVID-19 vaccine, whenever it becomes available. QUESTION: With reference to the Heeng (asafoetida) production in India, consider the following statements : 1. India is the largest producer of Heeng in the world. 2. It is one of the genetically modified (GM) crops produced in India. Select the correct codes given below: a) 1 only b) 2 only c) Both 1 and 2 d) Neither 1 nor 2 Correct answer: D Explanation: Both statements are incorrect. Heeng (asafoetida) is one of the widely used spices in Indian cuisine since time immemorial. But there is no production of heeng in India and currently about 1,200 tons of raw heeng worth Rs 600 crore is being imported annually from Afghanistan, Iran and Uzbekistan. CSIR Institute of Himalayan Bioresource Technology (CSIR-IHBT) has introduced six accessions of heeng from Iran through the National Bureau of Plant Genetic Resources (NBPGR), and standardized its production protocols under Indian conditions. Bt cotton is the only GM crop that is allowed in India. Heeng is a perennial plant and it produces oleo-gum resin from the roots after five years of plantation. It can be grown in unutilized sloppy land of cold desert regions. QUESTION: With reference to the Pariyojana, consider the following statements: 1. It focuses on optimizing efficiency of freight and passenger movement across the country. 2. It aims to promote port-led development in the country by harnessing India's coastline. Select the correct codes given below: a) 1 only b) 2 only c) Both 1 and 2 d) Neither 1 nor 2 Correct answer: A Explanation: Statement 1 is correct. Bharatmala Pariyojana is an umbrella program for the highways sector that focuses on optimizing efficiency of freight and passenger movement across the country by bridging critical infrastructure gaps through effective interventions like development of Economic Corridors, Inter Corridors and Feeder Routes, National Corridor Efficiency Improvement, Border and International connectivity roads, Coastal and Port connectivity roads and Green-field expressways. A multi-modal logistic park is being developed in under the Bharatmala Pariyojna. Statement 2 is incorrect. The Sagarmala programme aims to promote port-led development in the country through harnessing India's 7,500 km long coastline, 14500 km of potentially navigable waterways and strategic location on key international maritime trade routes. QUESTION: The 'Future of Jobs Report 2020' has been published by: a) World Economic Forum b) World Bank c) Organisation for Economic Co-operation and Development d) International Labour Organization Correct answer: A Explanation: The Future of Jobs report by World Economic Forum maps the jobs and skills of the future, tracking the pace of change. The report’s key findings include: --The pace of technology adoption is expected to remain unabated and may accelerate in some areas. --Automation, in tandem with the COVID-19 recession, is creating a ‘double-disruption’ scenario for workers. In addition to the current disruption from the -induced lockdowns and economic contraction, technological adoption by companies will transform tasks, jobs and skills by 2025. --Although the number of jobs destroyed will be surpassed by the number of ‘jobs of tomorrow’ created, in contrast to previous years, job creation is slowing while job destruction accelerates. --In the absence of proactive efforts, inequality is likely to be exacerbated by the dual impact of technology and the pandemic recession. QUESTION: With reference to the UDAN (Ude Desh ka Aam Nagrik) scheme, Consider the following statements: 1. The Airports Authority of India (AAI) is the Implementing Agency for the Scheme. 2. It provides Viability Gap Funding to selected airline operators for passenger as well as cargo operations. Select the correct codes given below: a) 1 only b) 2 only c) Both 1 and 2 d) Neither 1 nor 2 Correct answer: A Explanation: Statement 1 is correct. RCS-UDAN, "Ude Desh ka Aam Nagrik" is a 's flagship program which aims to provide affordable, economically viable and profitable air travel on regional routes. This Regional Connectivity Scheme offers unique opportunity to a common man to fly at an affordable price. The Airports Authority of India (AAI) has been designated as the Implementing Agency under this Scheme. Statement 2 is incorrect. Under this Scheme, support is provided to Selected Airline Operator(s) in the form of Viability Gap Funding and other concessions / support offered by the Central Government, State Governments and airport operators but the VGF support is not provided for cargo operations.

UDAN has played a major role in adding new airports and routes to the aviation landscape of the country. 50 unserved and underserved airports (including 5 heliports) with 285 routes have been added under UDAN across the length and breadth of India. AAI has an ambitious plan to develop at least 100 airports/waterdromes/heliports by 2024 under the Scheme.

QUESTION: With reference to the National Strategy for Financial Education (NSFE): 2020-2025, consider the following statements: 1. It has been prepared by NITI Aayog.

2. It has recommended a ‘5 C’ approach for dissemination of financial education in the country.

Which of the statements given above is/are correct?

A. 1 only

B. 2 only

C. Both 1 and 2 D. Neither 1 nor 2

Answer: B

The National Strategy for Financial Education (NSFE): 2020-2025 document was released by the Reserve Bank of India (RBI).

This NSFE for the period 2020-2025 is the second one after the 2013-18 NSFE.

It has been prepared by the National Centre for Financial Education (NCFE) in consultation with all the Financial Sector Regulators (RBI, SEBI, IRDAI and PFRDA), DFS and other Ministries of Govt. of India and other stakeholders under the aegis of the Technical Group on Financial Inclusion and Financial Literacy under the Chairmanship of Deputy Governor, RBI.

The document has been approved by the FSDC-SC in its 24th meeting, held on June 2020.

It has recommended a ‘5 C’ approach for dissemination of financial education in the country. These include emphasis on development of relevant content in curriculum in schools, colleges and training establishments, developing capacity among intermediaries involved in providing financial services, leveraging the positive effect of community-led model for financial literacy through appropriate communication strategy, and, enhancing collaboration among various stakeholders.

Hence only statement 2 is correct.

QUESTION: With reference to the Nationally Determined Contributions (NDC)–Transport Initiative for Asia (TIA), consider the following statements:

1. Aim of NDC–TIA programme is to promote a comprehensive approach to decarbonize transport in India, Vietnam, and China.

2. NDC–TIA is a joint programme, supported by the International Climate Initiative (IKI) of the German Ministry for the Environment, Nature Conservation and Nuclear Safety (BMU).

Which of the statements given above is/are correct?

A. 1 only

B. 2 only

C. Both 1 and 2

D. Neither 1 nor 2

Answer:C

NITI Aayog will launch the India Component of the Nationally Determined Contributions (NDC)– Transport Initiative for Asia (TIA) on 27 August.

 Aim of NDC–TIA programme is to promote a comprehensive approach to decarbonize transport in India, Vietnam, and China.  NDC–TIA is a joint programme, supported by the International Climate Initiative (IKI) of the German Ministry for the Environment, Nature Conservation and Nuclear Safety (BMU) and implemented by a consortium of seven organisations, namely:

o Deutsche Gesellschaft für Internationale Zusammenarbeit (GIZ) GmbH

o International Council on Clean Transportation (ICCT)

o World Resources Institute (WRI)

o International Transport Forum (ITF)

o Agora Verkehrswende (AGORA)

o Partnership on Sustainable, Low Carbon Transport (SLoCaT) Foundation

o Renewable Energy Policy Network for the 21st Century e.V. (REN21)

 The India Component is implemented by six consortium organisations, all except SLoCaT. On behalf of the Government of India, NITI Aayog, the country’s premier policy think tank, will be the implementing partner.

 The NDC-TIA programme has a duration of 4 years.

 The NDC–TIA India Component will focus on

o strengthening GHG and transport modelling capacities,

o providing technical support on GHG emission reduction measures,

o financing climate actions in transport,

o offering policy recommendations on electric vehicle (EV) demand and supply policies etc.

 Hence both statements are correct.

QUESTION: “KIRAN” helpline 1800-599-0019 was developed by which of the following union Ministry?

A. Ministry of Agriculture and Farmers Welfare

B. Ministry of Development of North Eastern Region

C. Ministry of Housing and Urban Affairs

D. Ministry of Social Justice and Empowerment

Answer: D

The Ministry of Social Justice & Empowerment (M/o SJ&E) has postponed the launch of “KIRAN” TOLL- FREE Mental Health Rehabilitation Helpline (1800-599-0019) number which was due to be launched on 27-08-2020.  The helpline 1800-599-0019, developed by the Social Justice and Empowerment Ministry, along with its partners aims at providing support for early screening, first-aid, psychological support, distress management, mental wellbeing, preventing deviant behaviour and psychological crisis management.

 The helpline will offer support in 13 languages for any individual, family, NGOs, DPOs, parent associations, professional associations, rehabilitation institutes, hospitals or anyone in need of support across the country.

 The helpline will be supported by 660 volunteers, clinical and rehabilitation psychologists and 668 volunteer psychiatrists along with 75 experts at 25 helpline centres which will have a capacity of handling 300 clients per hour.

 The helpline will be coordinated by the National Institute for the Empowerment of Persons with Multiple Disabilities (NIEPMD, Chennai) and the National Institute of Mental Health Rehabilitation (NIMHR, Sehore).

Hence, option (d) is the correct answer.

QUESTION: V. Ramagopal Rao committee, recently seen in news, is related to:

review the charter of duties for all laboratories of the Defence Research and Development A. Organisation (DRDO).

B. report on the working of capital market infrastructure institutions (MIIs)

C. decide the appropriate level of reserves that the RBI should hold

D. review the Operation of the Cash Credit System

Answer: A

With a focus on indigenous development of futuristic technologies, a five-member expert committee has been constituted to review the charter of duties for all laboratories of the Defence Research and Development Organisation (DRDO).

 The committee was constituted by G. Satheesh Reddy, Secretary, Department of Defence Research and Development (DDR&D), who is also the Chairman, DRDO.

 The five-member committee is headed by Professor V. Ramagopal Rao, Director, Indian Institute of Technology, Delhi.

 The terms of reference are: to study and review the charter of duties of all the labs of the DRDO, to redefine the charter of duties of the labs on the current and futuristic defence and battlefield scenario, and to minimise the overlap of technologies amongst the labs.

 The committee has to submit its report within 45 days.

Hence, option (a) is the correct answer

QUESTION: With reference to the International Day of Remembrance and Tribute to the Victims of Terrorism, consider the following statements:

1. International Day of Remembrance and Tribute to the Victims of Terrorism was observed on 21 August.

2. It was proclaimed to honour and support the victims and survivors of terrorism and to promote and protect the full enjoyment of their human rights and fundamental freedoms.

Which of the statements given above is/are correct?

A. 1 only

B. 2 only

C. Both 1 and 2

D. Neither 1 nor 2

Answer: C

International Day of Remembrance and Tribute to the Victims of Terrorism was observed on 21 August.

 The General Assembly, in its resolution 72/165 (2017), established 21 August as the International Day of Remembrance of and Tribute to the Victims of Terrorism.

 It was proclaimed to honour and support the victims and survivors of terrorism and to promote and protect the full enjoyment of their human rights and fundamental freedoms.

 The Global Counter-Terrorism Strategy, adopted unanimously in its resolution 60/288, on 8 September 2006, notes that the dehumanization of victims counts among the conditions conducive to the spread of terrorism.

 Hence both statements are correct.

QUESTION: With reference to the RLF-100 (Aviptadil), consider the following statements:

1. It is a formulation of synthetic human Vasoactive Intestinal Polypeptide (VIP).

2. Vasoactive Intestinal Polypeptide (VIP) is released only in lungs.

Which of the statements given above is/are correct?

A. 1 only

B. 2 only

C. Both 1 and 2

D. Neither 1 nor 2

Answer: A An old drug formulation — RLF-100 — is being probed to treat Covid-19 patients after initial studies have shown that the drug is able to halt SARS-CoV-2 virus replication in lungs.

 RLF-100, also called Aviptadil, is a formulation of synthetic human Vasoactive Intestinal Polypeptide (VIP).

 VIP is released throughout the body, but remains mostly concentrated in lungs.

 It is produced by immune cells and nerve endings and acts as a neurotransmitter. It helps improve muscle activity and blood flow in gastrointestinal tract. Studies have shown VIP has anti-inflammatory and anti-cytokine activity properties.

 RLF-100 is not a new finding. It was discovered in 1970 by Dr Sami Said, a pulmonary specialist.

 Hence only statement 1 is correct.

QUESTION: Consider the following statements:

1. India and Israel signed a cultural agreement that outlines a three-year programme of cooperation to further strengthen their strategic bilateral relations.

2. The programme of cooperation between India and Israel for the years 2020-23 is based on the cultural agreement signed between them on May 18, 2016.

Which of the statements given above is/are correct?

A. 1 only

B. 2 only

C. Both 1 and 2

D. Neither 1 nor 2

Answer: A

India and Israel signed a cultural agreement that outlines a three-year programme of cooperation to further strengthen their strategic bilateral relations.

 The programme of cooperation between the two countries for the years 2020-23 is based on the cultural agreement signed between them on May 18, 1993, a little more than a year after they established full-fledged diplomatic relations.

 The major areas of cooperation identified to promote cultural ties include the exchange of culture and art experts, encouraging cooperation in the protection of cultural heritage and archaeology and organising literary fests and book fairs.

 It also includes student exchanges through scholarships, encouraging the participation of films and film-makers in each other's international film festivals, devising youth exchange programmes and encouraging sports-related interaction among youth. Hence only statement 1 is correct.

QUESTION: With reference to the Atal Bimit Vyakti Kalyna Yojna, consider the following statements:

1. Micro Units Development & Refinance Agency Ltd. (MUDRA) is implementing the Atal Bimit Vyakti Kalyna Yojna.

2. Recently the scheme got extended for one more year upto 30th June next year.

Which of the statements given above is/are correct?

A. 1 only

B. 2 only

C. Both 1 and 2

D. Neither 1 nor 2

Answer: B

The Employees' State Insurance Corporation (ESIC) relaxed norms to pay 50 per cent of average wages of three months as unemployment benefit between March 24 and December 31 this year in view of the COVID-19 pandemic.

 The ESIC took the decision in its 182nd meeting held yesterday under the Chairmanship of Union Labour and Employment Minister.

 ESIC is implementing the Atal Bimit Vyakti Kalyna Yojna under which unemployment benefit is paid to the workers covered under Employees' State Insurance (ESI) Scheme.

 The ESIC has decided to extend the scheme for one more year upto 30th June next year. It has been decided to relax the existing conditions and the amount of relief for workers who have lost employment during the Covid-19 pandemic period.

 The eligibility criteria for availing the relief has also been relaxed.

o The payment of relief has been enhanced to 50 per cent of average of wages from earlier 25 percent of average wages payable upto maximum 90 days of unemployment.

o Instead of the relief becoming payable 90 days after unemployment, it shall become due for payment after 30 days.

Hence only statement 2 is correct.

QUESTION: ‘Framework agreement’, recently seen in news, is related to:

A. Naga issue

B. Bru tribe displacement C. Indus water treaty

D. None of the above

Answer: A

Fresh hurdles have emerged in the road to peace in Nagaland.

 After a framework agreement was signed in 2015 between the Centre and the Isak-Muivah faction of the National Socialist Council of Nagalim, or the NSCN (I-M), the largest of the extremist groups in the peace process, there have been more than 100 rounds of talks.

 The latest involves the demand by the NSCN (I-M) to remove Nagaland Governor R.N. Ravi as the Centre’s interlocutor for the 23-year-old peace process and his alleged tweaking of the original framework agreement.

 On August 3, 2015, the Centre signed a framework agreement with the NSCN (I-M) to resolve the Naga issue, but both sides maintained secrecy about its contents.

 A few days ago, the NSCN (I-M) released the contents of the framework agreement.

 The outfit said Mr. Ravi had “craftily deleted the word ‘new’ from the original” line that referred to “shared sovereignty” between India and the Naga homeland and provided for an “enduring inclusive new relationship of peaceful co-existence”.

 The NSCN (I-M) claimed “new” was a politically sensitive word that defined the meaning of peaceful co-existence of the two entities (sovereign powers) and strongly indicated a settlement outside the purview of the Constitution of India.

QUESTION: Which of the following statement is Not correct with respect to Mali?

A. Mali is a landlocked country in Horn of Africa.

B. Most of the country lies in the southern Sahara Desert.

C. It is the third largest producer of gold in the African continent and salt.

D. Country's southern part features the Niger and Senegal rivers.

Answer: A

Mali President Ibrahim Boubacar Keita has announced his resignation amid a military coup. The development comes after more than two months of regular demonstrations calling for him to step down three years before his final term was due to end.

 Mali is a landlocked country in West Africa. Its capital is Bamako.

 Mali borders Algeria to the north-northeast, Niger to the east, Burkina Faso to the south-east, Ivory Coast to the south, Guinea to the south-west, and Senegal to the west and Mauritania to the north-west.  Most of the country lies in the southern Sahara Desert. its borders on the north reach deep into the middle of the Sahara Desert, while the country's southern part features the Niger and Senegal rivers.

 Some of Mali's prominent natural resources include gold, being the third largest producer of gold in the African continent and salt.

 Hence, option (a) is the correct answer.

QUESTION: With reference to the World Humanitarian Day (WHD), consider the following statements:

1. On August 19, the World Humanitarian Day (WHD) is being celebrated to commemorate humanitarian workers killed and injured in the course of their work.

2. This year World Humanitarian Day is being celebrated with the theme “#RealLifeHeroes” paying special tribute to the real-life heroes who have committed their lives to helping others in the most extreme circumstances throughout the world.

Which of the statements given above is/are correct?

A. 1 only

B. 2 only

C. Both 1 and 2

D. Neither 1 nor 2

Answer: C

On August 19, the World Humanitarian Day (WHD) is being celebrated to commemorate humanitarian workers killed and injured in the course of their work.

 This day was designated in memory of the 19 August 2003 bomb attack on the Canal Hotel in Baghdad, Iraq, killing 22 people, including the chief humanitarian in Iraq, Sergio Vieira de Mello.

 In 2009, the United Nations General Assembly formalized the day as World Humanitarian Day.

 This year World Humanitarian Day is being celebrated with the theme “#RealLifeHeroes” paying special tribute to the real-life heroes who have committed their lives to helping others in the most extreme circumstances throughout the world.

 Hence both statements are correct.

QUESTION: With reference to the Integrated Regional Offices (IROs) of the MoEF&CC, consider the following statements:

1. Ministry of Environment, Forest and Climate Change (MoEF&CC) has recently approved establishment of 19 Integrated Regional Offices (IROs).

2. The head of each of the IRO will be called “Regional Officer” of MoEF&CC. Which of the statements given above is/are correct?

A. 1 only

B. 2 only

C. Both 1 and 2

D. Neither 1 nor 2

Answer: C

Ministry of Environment, Forest and Climate Change (MoEF&CC) has approved establishment of 19 Integrated Regional Offices (IROs).

 This has been done to achieve outcomes related to the mandates of Ministry in an improved manner and to further enhance its outreach to stakeholders.

 These IROs of the MoEF&CC will start functioning from October 1st, 2020.

 Each IRO shall have representation from existing Regional Office/Regional Centre of MoEF&CC, Forest Survey of India (FSI), National Tiger Conservation Authority (NTCA), Central Zoo Authority (CZA) and Wildlife Crime Control Bureau (WCCB) as available to them from time to time.

 The head of each of the IRO will be called “Regional Officer” of MoEF&CC.

 Hence both statements are correct

QUESTION: What is Dhanwantari Rath, recently seen in news?

A. A Rath yatra in Odisha established by Chodaganga Deva of the Ganga dynasty.

B. Mobile unit of Ayurveda services

C. India’s new anti-tank guided missile

D. None of the above

Answer: B

A MoU was signed between All India Institute of Ayurveda (AIIA) and Delhi Police for extending the Ayurveda Preventive and Promotive health services in the residential colonies of Delhi Police.

 These services are to be provided through a mobile unit named ‘Dhanwantari Rath’ and Police Wellness Centres and are to be catered by AIIA, supported by Ministry of AYUSH.

 Dhanwantari Rath – Mobile unit of Ayurveda health care services would consist a team of Doctors who would be visiting Delhi Police colonies regularly.  These Ayurveda Health care services are expected to reduce the incidence/prevalence of various diseases and also reduce the number of referrals to hospitals thereby reducing cost to healthcare system as well as patient.

 AYURAKSHA a joint venture of AIIA, an autonomous Institute under Ministry of AYUSH and Delhi Police aims for maintaining the health of frontline COVID warriors like Delhi police personal through Ayurveda immunity boosting measures.

 In continuation of the project, Ayurveda Preventive and Promotive health care are now planned to be extended to the families of Delhi Police personnel.

 Hence, option (b) is the correct answer.

QUESTION: With reference to ‘SRIJAN’ portal, consider the following statements:

1. It is an initiative of Department of Commerce, Govt of India.

2. It is a one stop shop online portal that provides access to the vendors to take up items that can be taken up for indigenization.

Which of the statements given above is/are correct?

A. 1 only

B. 2 only

C. Both 1 and 2

D. Neither 1 nor 2

Answer:B

Defence Minister Rajnath Singh has launched ‘SRIJAN’ portal which is a one stop shop online portal that provides access to the vendors to take up items that can be taken up for indigenization. Pursuant to Atmanirbhar Bharat announcement, Department of Defence Production has developed an indigenization portal, srijandefence.gov.in, as 'opportunities for ' in Defence, which will give information on items that can be taken up for indigenization by the private sector. On this portal, DPSUs/OFB/SHQs can display their items which they have been importing or are going to import which the Indian Industry can design, develop and manufacture as per their capability or through joint venture with OEMs. The Indian Industry will be able to show their interest. The concerned DPSUs/OFB/SHQs, based on their requirement of the items and their guidelines & procedures will interact with the Indian industry for indigenization. Hence only statement 2 is correct.

QUESTION: Recently initiative, Krishi Megh is aimed at protecting the precious data of which of the following organisations?

A. CSIR

B. DRDO C. Indian Council of Agricultural Research (ICAR)

D. None of the above

Answer: C

Union Agriculture Minister has launched ICAR's data recovery centre - Krishi Megh. The move is aimed at protecting the precious data of the government's premier research body Indian Council of Agricultural Research (ICAR). It has been set up at National Academy of Agricultural Research Management (NAARM) in Hyderabad. NAARM, Hyderabad has been chosen as it lies in a different seismic zone with regard to the Data Centre at ICAR-IASRI in New Delhi. Currently, the main data centre of the ICAR is at the Indian Agricultural Statistics Research Institute (IASRI) in the national capital. Krishi Megh has been set up under the National Agricultural Higher Education Project (NAHEP), funded by both the government and World Bank. Hence, option (c) is the correct answer.

QUESTION: Which of the following state government has introduced ‘Samadhan-se-vikas’, a one-time settlement scheme for recovery of External Development Charges (EDC)?

A.

B. Kerala

C.

D. Gujarat

Answer:A

Recently, the Haryana government has introduced ‘Samadhan-se-vikas’, a one-time settlement scheme for recovery of External Development Charges (EDC). The new scheme called ‘Samadhan se Vikas’ is modelled on the central scheme of ‘Vivad se Vishwas-2020’. The scheme will be applicable to the full outstanding EDC including interest as well as penal interest. Several real estate giants in Haryana have not deposited hundreds of crores of rupees worth mandatory EDC for the residential and commercial colonies they have built across Haryana. In a bid to recover this massive sum, Haryana government introduced this scheme. The developer is supposed to pay EDC to civic authorities for maintenance of civic amenities within the periphery of the developed project including construction of roads, water and electricity supply, landscaping, maintenance of drainage and sewage systems, waste management etc. The EDC is decided by the civic authorities. Hence, option (a) is the correct answer.

QUESTION: With reference to the Next Generation Sequencing machines (NGS), consider the following statements:

1. It is an initiative of Council of Scientific and Industrial Research (CSIR).

2. It aims to ramp up testing for COVID-19 as well as improve the accuracy rate.

Which of the statements given above is/are correct? A. 1 only

B. 2 only

C. Both 1 and 2

D. Neither 1 nor 2

Answer: C

The Council of Scientific and Industrial Research (CSIR) is working on developing “mega labs” to ramp up testing for COVID-19 as well as improve the accuracy rate. The labs will be repurposing large machines, called Next Generation Sequencing machines (NGS), which are normally used for sequencing human genomes, to sequence 1,500 to 3,000 viral genomes at a go to detect the SARS- CoV-2 virus. The CSIR has partnered with the U.S.-based Illumina, a company that specialises in the manufacture of NGS machines. Five such sequencers, costing ₹4 crore each, are currently available in India. These machines can substantially detect the presence of the virus even in several instances where the traditional RT-PCR (reverse transcription polymerase chain reaction) tests fail. This is because the RT-PCR test identifies the SARS-CoV-2 virus by exploring only specific sections, whereas the genome method can read a bigger chunk of virus genome. It can also trace the evolutionary history of the virus and track mutations more reliably. Unlike the RT-PCR that needs primers and probes the NGS only needs custom reagents. Hence both statements are correct.

QUESTION: What is AR2770, recently seen in news?

A. A massive Sunspot group

B. An exoplanet

C. A group of malware

D. None of the above

Answer: A

A massive Sunspot group, AR2770, was observed recently by spaceweather.com using images of the Sun’s surface from NASA’s Solar Dynamics Observatory (SDO). A Sunspot is an area on the Sun that appears dark on the surface and is relatively cooler than surrounding parts. These spots, some as large as 50,000 km in diameter, are the visible markers of the Sun’s magnetic field, which forms a blanket that protects the solar system from harmful cosmic radiation. On the photosphere– the outer surface of the Sun which radiates heat and light– Sunspots are the areas where the star’s magnetic field is the strongest; around 2,500 times more than the Earth’s magnetic field. Most Sunspots appear in groups that have their own magnetic field, whose polarity reverses during every solar cycle, which takes around 11 years. In every such cycle, the number of Sunspots increases and decreases. The current solar cycle, which began in 2008, is in its ‘solar minimum’ phase, when the number of Sunspots and solar flares is at a routine low. Hence, option (a) is the correct answer.

QUESTION: With reference to the International Day of the World’s Indigenous People, consider the following statements:

1. It is being observed on August 9.

2. This year’s theme is “COVID-19 and indigenous peoples’ resilience”.

Which of the statements given above is/are incorrect?

A. 1 only

B. 2 only

C. Both 1 and 2

D. Neither 1 nor 2

Answer: D

International Day of the World’s Indigenous Peoples or World Tribal Day is being observed on August 9. This year’s theme is “COVID-19 and indigenous peoples’ resilience”. The day is aimed at promoting and protecting the rights of the world’s indigenous population. The date recognizes the first meeting of the United Nations Working Group on Indigenous Populations in Geneva in 1982. Hence both statements are correct.

QUESTION: With reference to the Khadi Agarbatti Aatmanirbhar Mission, consider the following statements:

1. The program named as “Khadi Agarbatti Aatmanirbhar Mission” aims at creating employment for unemployed and migrant workers in different parts of the country while increasing domestic Agarbatti production substantially.

2. Khadi and Village Industries Commission will provide 25% subsidy on the cost of the machines and will recover the remaining 75% of the cost from the artisans in easy installments every month.

Which of the statements given above is/are correct?

A. 1 only

B. 2 only

C. Both 1 and 2

D. Neither 1 nor 2

Answer: C

Union Minister for MSME has approved a unique employment generation program proposed by Khadi and Village Industries Commission (KVIC) to make India Aatmanirbhar in Agarbatti production.  The program named as “Khadi Agarbatti Aatmanirbhar Mission” aims at creating employment for unemployed and migrant workers in different parts of the country while increasing domestic Agarbatti production substantially. The pilot project will be launched soon.

 The scheme is designed by KVIC on PPP mode. Under the scheme, KVIC will provide Automatic Agarbatti making machines and powder mixing machines to the artisans through the successful private Agarbatti manufacturers who will sign the agreement as business partners.

 KVIC will provide 25% subsidy on the cost of the machines and will recover the remaining 75% of the cost from the artisans in easy installments every month. The business partner will provide the raw material to the artisans for making Agarbatti and will pay them wages on job work basis.

 Cost of artisans’ training will be shared between KVIC and the private business partner wherein KVIC will bear 75% of the cost while 25% will be paid by the business partner.

 The current consumption of Agarbatti in the country is approximately 1490 MT per day; however, India’s per day production of Agarbatti is just 760 MT. There is a huge gap between the demand and the supply and hence, immense scope for job creation.

Hence both statements are correct.

QUESTION: With reference to the ‘Tabletop Airport’, consider the following statements:

1. It is an airport located and built on top of a plateau or hilly surface, with one or both ends of the runway overlooking a drop.

2. Cochin International airport is a Tabletop Airport.

Which of the statements given above is/are correct?

A. 1 only

B. 2 only

C. Both 1 and 2

D. Neither 1 nor 2

Answer:A

On 7 August 2020, a Boeing 737 aircraft of Air India Express (the low cost subsidiary of national carrier Air India) overshot the runway while landing at Calicut International Airport. The crash killed 18 people and over 100 were injured. The accident has once again turned the spotlight on operations to what are called ‘tabletop airports’ in India. Tabletop Airport’ is an airport located and built on top of a plateau or hilly surface, with one or both ends of the runway overlooking a drop. There are not many differences between a ‘normal’ airport and a ‘tabletop’ airport. The airports in the country which would count as “tabletops”, are namely Lengpui (Mizoram), Shimla and Kullu (), Pakyong (Sikkim), Mangaluru (Karnataka), Kozhikode and Kannur (both Kerala). There is no such term as a ‘tabletop airport’ in any International Civil Aviation Organisation (ICAO) technical document. But India’s statutory aviation body, the Directorate General of Civil Aviation (DGCA), refers to these airports in this manner by way of highlighting safety measures during operations to these runways. Hence, option (a) is the correct answer.

QUESTION: With reference to the Emergency Credit Line Guarantee Scheme, consider the following statements: 1. The Emergency Credit Line Guarantee Scheme was rolled out as part of the Centre’s Aatmanirbhar package in response to the COVID-19 crisis.

2. It has a corpus of ₹41,600 crore and provides fully guaranteed additional funding of up to ₹3 lakh crore.

Which of the statements given above is/are incorrect?

A. 1 only

B. 2 only

C. Both 1 and 2

D. Neither 1 nor 2

Answer: D

The Centre has expanded its credit guarantee scheme for micro, small and medium enterprises (MSMEs) to cover loans given to larger firms, as well as to self-employed people and professionals who have taken loans for business purposes. The Emergency Credit Line Guarantee Scheme was rolled out in May as part of the Centre’s Aatmanirbhar package in response to the COVID-19 crisis. It has a corpus of ₹41,600 crore and provides fully guaranteed additional funding of up to ₹3 lakh crore. Eligible MSMEs had to have an annual turnover up to ₹100 crore, with outstanding loans of up to ₹25 crore as on February 29, 2020. The scheme has been expanded to cover enterprises with a turnover up to ₹250 crore, with outstanding loans up to ₹50 crore. Individual beneficiaries include both professionals such as doctors, lawyers and chartered accountants, as well as self-employed people such as vendors or taxi drivers. Hence both statements are correct.

QUESTION: Who among the following was recently conferred with the Tamil Nadu chief minister's special award, in recognition of her advisory role in combating the Covid-19 pandemic in the state?

A. Soumya Swaminathan

B. Ritu Karidhal Srivastava

C. Devi Prasad Shetty

D.

Answer: A Chief Scientist of World Health Organisation, Soumya Swaminathan was conferred with the Tamil Nadu chief minister's special award, in recognition of her advisory role in combating the Covid-19 pandemic in the state. The WHO representative has been offering suggestions to the state health department to step up measures in fighting the pandemic. According to the World Health Organisation, Swaminathan has 30 years of experience in clinical care and research and has worked throughout her career to translate her research into impactful programmes. Hence, option (a) is the correct answer.

QUESTION: Which of the following became the first Central Armed Police Force (CAPF) to have supplies from the Khadi & Village Industries Commission, KVIC?

A. SSB

B. ITBP

C. CRPF

D. CISF

Answer:B

Indo Tibetan Border Police, ITBP became the first Central Armed Police Force CAPF to have supplies from the Khadi & Village Industries Commission, KVIC. An agreement was signed between the ITBP and the KVIC in New Delhi. According to the agreement, total 1200 Quintal of Mustard Oil is being procured from KVIC by ITBP with a total financial implication of 1 Crore 73 Lakh 80 Thousand rupees. It was decided during a meeting of the Directors General of the CAPFs held at Home Ministry in October last year that use of Terry Khadi Uniform and other items of swadeshi origin should be made available to the CAPFs. The ITBP had suggested that Durrie, Blankets, Towel, Mustard Oil, Yoga Kit, Hospital Bed Sheets, Pickles etc can be purchased for jawans of the Force through KVIC. Hence, option (b) is the correct answer.

QUESTION: With reference to the Kisan Special Parcel Train, consider the following statements:

1. The country's first Kisan Special Parcel Train or Kisan Rail will start from August 7, 2020 to provide seamless supply of perishable produce.

2. The train will transport material between 's Devlali and 's Danapur Railway station.

Which of the statements given above is/are correct?

A. 1 only

B. 2 only

C. Both 1 and 2 D. Neither 1 nor 2

Answer:C

The country's first Kisan Special Parcel Train or Kisan Rail will start from August 7, 2020 to provide seamless supply of perishable produce. The train will transport material between Maharashtra's Devlali and Bihar's Danapur Railway station. It will covering a distance of 1,519 kilometres in around 32 hours. The Kisan Rail will carry fruits and vegetables and will make stoppages at several stations and pick-up and deliver them. This will help in bringing perishable agricultural products like vegetables, fruits to the market in a short period of time. The Kisan Rail train with frozen containers is expected to build a seamless national cold supply chain for perishables, inclusive of fish, meat and milk. Finance Minister Nirmala Sitharaman had announced to start 'Kisan Rail' in the current year's Budget, for providing a seamless supply chain of perishable produce. Hence both statements are correct.

QUESTION: With reference to the Emergency Credit Line Guarantee Scheme, consider the following statements: 1. The Emergency Credit Line Guarantee Scheme was rolled out as part of the Centre’s Aatmanirbhar package in response to the COVID-19 crisis.

2. It has a corpus of ₹41,600 crore and provides fully guaranteed additional funding of up to ₹3 lakh crore.

Which of the statements given above is/are incorrect?

A. 1 only

B. 2 only

C. Both 1 and 2

D. Neither 1 nor 2

Answer: D

The Centre has expanded its credit guarantee scheme for micro, small and medium enterprises (MSMEs) to cover loans given to larger firms, as well as to self-employed people and professionals who have taken loans for business purposes. The Emergency Credit Line Guarantee Scheme was rolled out in May as part of the Centre’s Aatmanirbhar package in response to the COVID-19 crisis. It has a corpus of ₹41,600 crore and provides fully guaranteed additional funding of up to ₹3 lakh crore. Eligible MSMEs had to have an annual turnover up to ₹100 crore, with outstanding loans of up to ₹25 crore as on February 29, 2020. The scheme has been expanded to cover enterprises with a turnover up to ₹250 crore, with outstanding loans up to ₹50 crore. Individual beneficiaries include both professionals such as doctors, lawyers and chartered accountants, as well as self-employed people such as vendors or taxi drivers. Hence both statements are correct.

QUESTION: Who among the following was recently conferred with the Tamil Nadu chief minister's special award, in recognition of her advisory role in combating the Covid-19 pandemic in the state? A. Soumya Swaminathan

B. Ritu Karidhal Srivastava

C. Devi Prasad Shetty

D. Neelam Kler

Answer: A

Chief Scientist of World Health Organisation, Soumya Swaminathan was conferred with the Tamil Nadu chief minister's special award, in recognition of her advisory role in combating the Covid-19 pandemic in the state. The WHO representative has been offering suggestions to the state health department to step up measures in fighting the pandemic. According to the World Health Organisation, Swaminathan has 30 years of experience in clinical care and research and has worked throughout her career to translate her research into impactful programmes. Hence, option (a) is the correct answer.

QUESTION: Which of the following became the first Central Armed Police Force (CAPF) to have supplies from the Khadi & Village Industries Commission, KVIC?

A. SSB

B. ITBP

C. CRPF

D. CISF

Answer:B

Indo Tibetan Border Police, ITBP became the first Central Armed Police Force CAPF to have supplies from the Khadi & Village Industries Commission, KVIC. An agreement was signed between the ITBP and the KVIC in New Delhi. According to the agreement, total 1200 Quintal of Mustard Oil is being procured from KVIC by ITBP with a total financial implication of 1 Crore 73 Lakh 80 Thousand rupees. It was decided during a meeting of the Directors General of the CAPFs held at Home Ministry in October last year that use of Terry Khadi Uniform and other items of swadeshi origin should be made available to the CAPFs. The ITBP had suggested that Durrie, Blankets, Towel, Mustard Oil, Yoga Kit, Hospital Bed Sheets, Pickles etc can be purchased for jawans of the Force through KVIC. Hence, option (b) is the correct answer.

QUESTION: With reference to the 'Sahakar Cooptube NCDC India', consider the following statements: 1. It aims to encourage farmers and the youth to take benefit of cooperatives.

2. National Cooperative Development Corporation (NCDC) is an apex-level statutory institution under the Ministry of Finance. Which of the statements given above is/are correct?

A. 1 only

B. 2 only

C. Both 1 and 2

D. Neither 1 nor 2

Answer: A

Union Agriculture Minister launched the National Cooperative Development Corporation's YouTube channel, 'Sahakar Cooptube NCDC India', to encourage farmers and the youth to take benefit of cooperatives. He also released videos on formation and registration of cooperatives for 18 states in Hindi and regional languages. The videos will be aired on the new channel. The guidance videos in different languages covering 18 states would also strengthen and deepen the major initiatives of our government to promote and form 10,000 farmer-producer organisations (FPOs). More states will be added to the collection of guidance videos on Sahakar Cooptube NCDC India channel on YouTube in due course of time. National Cooperative Development Corporation (NCDC) as an apex-level statutory institution under the Ministry of Agriculture and Farmers' Welfare has achieved tremendous success with a cumulative financial assistance to cooperatives to the tune of Rs 1,54,000 crore. Hence, option (a) is the correct answer.

QUESTION: ICAR-National Bureau of Plant Genetic Resources (NBPGR) is under which of the following union ministry?

A. Ministry of Agriculture

B. Ministry of AYUSH

C. Ministry of Science & Technology

D. Ministry of Environment and Forests

Answer:A

National Medicinal Plants Board (NMPB) and ICAR-National Bureau of Plant Genetic Resources (NBPGR) have entered into a MoU to conserve the Medicinal and Aromatic Plants Genetic Resources (MAPGRs). The conservation will be done at designated space of ICAR-NBPGR in long-term storage module in the National Gene bank and/or at Regional Station for medium term storage module. National Medicinal Plants Board (NMPB) is under Ministry of AYUSH. ICAR-National Bureau of Plant Genetic Resources (NBPGR) is under Department of Agricultural Research and Education. Hence, option (a) is the correct answer.

QUESTION: With reference to the Modern Slavery, consider the following statements: 1. According to a report by Commonwealth Human Rights Initiative (CHRI) and an international anti- slavery organisation ‘Walk Free’ Commonwealth countries accounting for about 40% of people living in conditions of modern slavery in the world.

2. India, like all other Commonwealth countries in Asia, had not ratified the International Labour Organisation’s 2011 Domestic Workers Convention or the 2014 Forced Labour Protocol.

Which of the statements given above is/are correct?

A. 1 only

B. 2 only

C. Both 1 and 2

D. Neither 1 nor 2

Answer: C

On the occasion of World Day Against Trafficking in Persons, the Commonwealth Human Rights Initiative (CHRI) and an international anti-slavery organisation ‘Walk Free’, released a report on Modern Slavery.

The report assessed the progress made by Commonwealth countries on the promises made in 2018 to end modern slavery by 2030 and achieve the Sustainable Development Goal of ending forced labour, human trafficking and child labour.

Commonwealth countries accounting for about 40% of people living in conditions of modern slavery in the world. An estimated one in every 150 people in the Commonwealth is living in conditions of modern slavery.

One-third of the Commonwealth countries had criminalised forced marriage, while 23 had not criminalised commercial sexual exploitation of children.

Out of 54 countries, only four engage with business to investigate supply chains, and all countries report gaps in victim assistance programs.

India, like all other Commonwealth countries in Asia, had not ratified the International Labour Organisation’s 2011 Domestic Workers Convention or the 2014 Forced Labour Protocol.

India accounted for one-third of all child brides in the world.

India has the weakest response on national coordination, with no national coordinating body or National Action Plan in place”.

Hence both statements are correct.

QUESTION: With reference to the Prem Bhatia award for outstanding journalism, consider the following statements:

1. The trust had instituted the awards in 1995 in the memory of journalist Prem Bhatia. 2. Dipankar Ghose and the People’s Archive of Rural India (PARI), a non-profit journalism website dedicated to reporting on rural India, has won this year’s Prem Bhatia award for outstanding journalism.

Which of the statements given above is/are correct?

A. 1 only

B. 2 only

C. Both 1 and 2

D. Neither 1 nor 2

Answer: C

Indian Express reporter Dipankar Ghose and the People’s Archive of Rural India (PARI), a non-profit journalism website dedicated to reporting on rural India, has won this year’s Prem Bhatia award for outstanding journalism. The Prem Bhatia Memorial Trust confers the awards for outstanding journalism. The trust had instituted the awards in 1995 in the memory of journalist Prem Bhatia, who died that year. The main objective of the Trust is to propagate the values of objective reporting, fearless pursuit of the truth and a commitment to improving the standards of journalism in India. Outstanding Political Reporting of The Year: An annual award of INR 2,00,000/- is given to an outstanding journalist who has reported the major political events of the year with objectivity. Outstanding Environmental Reporting of The Year: An annual award of INR.1,50,000/- is awarded for the journalist who has shown most promise in reporting on environmental, developmental or social affairs. Hence both statements are correct.

QUESTION: With reference to the renewable energy, consider the following statements:

1. India has set itself a target of 175 GW renewable energy capacity by 2022 including 100 GW of solar power and 60 GW of wind power capacity.

2. Solar Energy Corporation of India Ltd (SECI) is a CPSU under the administrative control of the Ministry of New and Renewable Energy (MNRE).

Which of the statements given above is/are correct?

A. 1 only

B. 2 only

C. Both 1 and 2

D. Neither 1 nor 2

Answer: C

Union Power Minister dedicated Sembcorp's state of the art SECI 1, 2 and 3 wind projects to the nation.

Sembcorp Energy India Limited (SEIL), a wholly-owned subsidiary of Sembcorp Industries, announced completion of its latest 800 MW wind power projects, bringing its India renewable energy capacity to 1,730 MW.

With the full commissioning of its 300MW SECI 3 wind project, Sembcorp becomes the first independent power producer to fully commission its projects awarded in the first three wind auctions held by the Solar Energy Corporation of India (SECI).

This capacity is also the largest operational wind capacity with any developer to-date from SECI auctions.

India has set itself a target of 175 GW renewable energy capacity by 2022 including 100 GW of solar power and 60 GW of wind power capacity.

Solar Energy Corporation of India Ltd (SECI) is a CPSU under the administrative control of the Ministry of New and Renewable Energy (MNRE). It was set up in 2011 and is a Section-3 company under the Companies Act, 2013. The mandate of the company has been broadened to cover the entire renewable energy domain.

QUESTION: With reference to the UN Economic Commission For Europe (UNECE), consider the following statements: 1. The United Nations Economic Commission for Europe (UNECE) was set up in 1947 by United Nations Economic and Social Council (ECOSOC).

2. It is one of five regional commissions of the United Nations.

3. It is headquartered in New York.

Which of the statements given above is/are correct?

A. 1 and 2 only

B. 2 and 3 only

C. 1 and 3 only

D. 1, 2 and 3

Answer: A

The Ministry of Road Transport and Highways has published a notification to amend Rule-93 relating to dimensions of motor vehicles under the Central Motor Vehicle Rules 1989. These amendments would provide for standardization in the dimensions of the Motor Vehicles which would be in line with international UNECE standards. It is a step by the Ministry to improve the logistics efficiency in the country as the enhanced dimensions would provide for extra passengers or extra carrying capacity within the prescribed weight. The United Nations Economic Commission for Europe (UNECE) was set up in 1947 by United Nations Economic and Social Council (ECOSOC). It is one of five regional commissions of the United Nations. UNECE's major aim is to promote pan-European economic integration. UNECE includes 56 member States in Europe, North America and Asia. It is headquartered in Geneva, Switzerland. Hence, option (a) is the correct answer.

QUESTION: With reference to the Union Public Service Commission, consider the following statements:

1. As per Article 316, the Chairman and other members of UPSC shall be appointed by the President.

2. As per Article 319, a person who holds office as Chairman shall, on the expiration of his term of office, be ineligible for re-appointment to that office.

Which of the statements given above is/are correct?

A. 1 only

B. 2 only

C. Both 1 and 2

D. Neither 1 nor 2

Answer: C

Pradeep Kumar Joshi, presently Member, Union Public Service Commission, took the oath of office and secrecy as Chairman, Union Public Service Commission (UPSC). He joined the Commission as Member in 2015. Prior to joining the Commission, he held the post of Chairman, Public Service Commission and Chairman, Public Service Commission. He also served as Director, National Institute of Educational Planning and Administration (NIEPA).

Union Public Service Commission (UPSC) is India's central recruiting agency. It is responsible for appointments to and examinations for All India services and group A & group B of Central services.

Formed in: 1926.

HQ: Delhi.

Appointment of Chairman:

As per Article 316, the Chairman and other members of UPSC shall be appointed by the President.

A member of a UPSC shall hold office for a term of six years from the date on which he enters upon his office or until he attains the age of 65 years, whichever is earlier.

As per Article 319, a person who holds office as Chairman shall, on the expiration of his term of office, be ineligible for re-appointment to that office. Hence only statement 1 is correct.

QUESTION: With reference to the Haryana state employment of local candidates ordinance, 2020, consider the following statements:

1. It reserves 75% private sector jobs for residents of the state.

2. The employers will have the option to recruit local candidates from one district to only 10 per cent.

Which of the statements given above is/are correct?

A. 1 only

B. 2 only

C. Both 1 and 2

D. Neither 1 nor 2

Answer: C

The Haryana Cabinet has cleared a proposal to draft an ordinance for reserving 75% private sector jobs for residents of the state. The draft ordinance will be placed before the Council of Ministers in its next meeting.

Under it, 75% of the new employment will be given to local candidates for jobs having salary of less than Rs 50,000 per month in various privately managed companies, Societies, Trusts, Limited Liability Partnership Firms, partnership firms etc. situated in the State of Haryana.

However, the employers will have the option to recruit local candidates from one district to only 10 per cent.

Exemption clause shall also be provided if suitable local candidates are not available for a particular category of industry.

Hence both statements are correct.

QUESTION: With reference to the draft Code on Wages (Central) Rules 2020, consider the following statements:

1. The normal working day shall comprise of “eight hours of work and one or more intervals of rest which in total shall not exceed one hour”.

2. The Centre shall constitute a technical committee which would advise on the skill categories, while an advisory board may recommend the minimum wage.

Which of the statements given above is/are correct? A. 1 only

B. 2 only

C. Both 1 and 2

D. Neither 1 nor 2

Answer: C

The Labour and Employment Ministry has notified the draft Code on Wages (Central) Rules under the Code on Wages, 2019 for public comments. The normal working day shall comprise of “eight hours of work and one or more intervals of rest which in total shall not exceed one hour”. Provisions of Factories Act will not be affected by this rule. It would fix a National Floor Minimum Wage based on factors like Net intake of 2,700 calories per day, 66 meters cloth per year per standard working class family, housing rent expenditure as 10 per cent of food and clothing expenditure, three adult consumption unit etc. The Centre shall constitute a technical committee which would advise on the skill categories, while an advisory board may recommend the minimum wage. Hence both statements are correct.

QUESTION: With reference to the SpaceX's Demo-2 Crew Dragon spacecraft, consider the following statements:

1. It was the first commercially built and operated spacecraft to carry people to and from orbit.

2. SpaceX's Demo-2 Crew Dragon spacecraft splashdown is the first water landing since 1975, when the NASA astronauts on the joint Apollo-Soyuz test mission returned home.

Which of the statements given above is/are correct?

A. 1 only

B. 2 only

C. Both 1 and 2

D. Neither 1 nor 2

Answer: C

SpaceX's Demo-2 Crew Dragon spacecraft successfully splashed down off the Pensacola, Florida coast in the Gulf of Mexico. After a two-month trip to International Space Station, the SpaceX Crew Dragon Endeavour carrying NASA astronauts Bob Behnken and Doug Hurley returned to Earth. It was the first splashdown by U.S. astronauts in 45 years, with the first commercially built and operated spacecraft to carry people to and from orbit. The return clears the way for another SpaceX crew launch as early as next month and possible tourist flights next year. The splashdown is the first water landing since 1975, when the NASA astronauts on the joint Apollo-Soyuz test mission returned home. Hence both statements are correct.

QUESTION: The Electronic Vaccine Intelligence Network (eVIN) is an innovative technological solution aimed at strengthening immunization supply chain systems across the country. It is being implemented under: A. Mission Parivar Vikas

B. National Health Mission

C. Janani Suraksha Yojana

D. Pradhan Mantri Surakshit Matritva Abhiyaan

Answer: B

Since April 2020, eight Indian States are using the eVIN application with 100 % adherence rate to track State specific COVID-19 material supplies, ensure availability and raise alerts in case of shortage of 81 essential drugs and equipment. The Electronic Vaccine Intelligence Network (eVIN) is an innovative technological solution aimed at strengthening immunization supply chain systems across the country. eVIN aims to provide real-time information on vaccine stocks and flows, and storage temperatures across all cold chain points in the country. This is being implemented under National Health Mission (NHM) by Ministry of Health and Family Welfare. eVIN has reached 32 States and Union Territories (UTs) and will soon be rolled-out in the remaining States and UTs of Andaman & Nicobar Islands, Chandigarh, Ladakh and Sikkim. Hence, option (b) is the correct answer.

QUESTION: With reference to FDI in the commercial coal mining, consider the following statements:

1. Recently, the government modified its FDI policy according to which any entity of a country, which shares land border with India or where the beneficial owner of an investment into India is situated in or is a citizen of any such country, could invest only under the Government route.

2. Government of India had earlier permitted 100 % FDI under automatic route in coal mining activities, including associated processing infrastructure for sale of coal.

Which of the statements given above is/are correct?

A. 1 only

B. 2 only

C. Both 1 and 2

D. Neither 1 nor 2

Answer: C

Government has clarified that any FDI in the commercial coal mining should also comply with the recent amendments made in the applicable laws. Recently, the government modified its FDI policy according to which any entity of a country, which shares land border with India or where the beneficial owner of an investment into India is situated in or is a citizen of any such country, could invest only under the Government route. The new policy further states that a citizen of Pakistan or an entity incorporated in Pakistan can invest, only under the Government route, in sectors and activities other than defence, space and atomic energy. Even for the sectors which are prohibited for foreign investment, Government route has been made mandatory for all such entities. Government of India had earlier permitted 100 % FDI under automatic route in coal mining activities, including associated processing infrastructure for sale of coal. Hence both statements are correct.

QUESTION: With reference to the Affordable Rental Housing Complexes (ARHC) Knowledge Pack, consider the following statements:

1. The Union Cabinet had approved ARHC as a sub scheme under Pradhan Mantri Awas Yojana (Urban) to provide housing facility to migrant workers and urban poor.

2. Under the first model of ARHC, existing Government funded vacant houses will be converted into ARHCs through Public Private Partnership or by public agencies for a period of 25 years.

Which of the statements given above is/are correct?

A. 1 only

B. 2 only

C. Both 1 and 2

D. Neither 1 nor 2

Answer: C

Union Housing Minister released Affordable Rental Housing Complexes (ARHC) Knowledge Pack. This includes MoUs to be signed with States and UTs to provide ease of living to urban migrants in the country. The Union Cabinet had approved ARHC as a sub scheme under Pradhan Mantri Awas Yojana (Urban) earlier this month to provide housing facility to migrant workers and urban poor. The Affordable Housing Complex scheme will be implemented through two models in the country. Under the first model existing Government funded vacant houses will be converted into ARHCs through Public Private Partnership or by public agencies for a period of 25 years. Under the second model ARHCs will be constructed, operated and maintained by Public or Private Entities on their own available vacant land for a period of 25 years. Successful implementation of ARHC Scheme will not only benefit urban migrants and poor but will also accelerate entrepreneurship and investment in rental housing market giving boost to the economy. Hence both statements are correct.

QUESTION: With reference to the Vidyarthi Vigyan Manthan, consider the following statements: 1. Vidyarthi Vigyan Manthan (VVM) is an initiative of Vijnana Bharati (VIBHA), in collaboration with NCERT and Vigyan Prasar.

2. It is a national program for popularizing result oriented research in Indian Institute of Technologies.

Which of the statements given above is/are correct?

A. 1 only

B. 2 only

C. Both 1 and 2

D. Neither 1 nor 2

Answer: A

Government of India launched 'Vidyarthi Vigyan Manthan, 2020-21. Vidyarthi Vigyan Manthan (VVM) is an initiative of Vijnana Bharati (VIBHA), in collaboration with Vigyan Prasar, an autonomous organization under the Department of Science and Technology, and NCERT, an institution under the Ministry of Education. VVM is a national program for popularizing science among school students of standard VI to XI, conceptualised to identify the bright minds with a scientific aptitude among the student community. Hence only statement 1 is correct.

QUESTION: With reference to the Rashtriya Swachhata Kendra, consider the following statements:

1. The RSK has been established at the Gandhi Smriti and Darshan Samiti at Rajghat in New Delhi.

2. RSK will impart information, awareness and education on the successful journey of world's largest behaviour change campaign, the .

Which of the statements given above is/are correct?

A. 1 only

B. 2 only

C. Both 1 and 2

D. Neither 1 nor 2

Answer: C

Prime Minister shall inaugurate Rashtriya Swachhata Kendra, an interactive experience centre on the Swachh Bharat Mission, on 8th August, 2020.

A tribute to Mahatma Gandhi, the Rashtriya Swachhata Kendra (RSK) was first announced by the Prime Minister in 2017, on the occasion of the centenary celebrations of Gandhiji's Champaran Satyagraha. The RSK has been established at the Gandhi Smriti and Darshan Samiti at Rajghat in New Delhi.

RSK will impart information, awareness and education on the successful journey of world's largest behaviour change campaign, the Swachh Bharat Mission.

The Swachh Bharat Mission has transformed rural sanitation in India and has changed the behaviours of over 55 crore people from open defecation to using a toilet.

The mission is now in its second phase, aiming to take India’s villages from Open Defecation Free (ODF) to ODF Plus, with a strong focus on sustaining ODF status and ensuring solid and liquid waste management for all.

Hence both statements are correct.

QUESTION: With reference to the Letter of Recommendation module for PM SVANidhi Scheme, consider the following statements: 1. The module is designed to give access to street vendors who do not have Identity Card and Certificate of Vending and are not in the surveyed list for availing benefits under the scheme.

2. It was launched by Union ministry of Finance.

Which of the statements given above is/are correct?

A. 1 only

B. 2 only

C. Both 1 and 2

D. Neither 1 nor 2

Answer: A

Union Urban Housing and Urban Affairs Ministry launched Letter of Recommendation module for PM SVANidhi Scheme.

The module is designed to give access to street vendors who do not have Identity Card and Certificate of Vending and are not in the surveyed list for availing benefits under the scheme.

The module provides for an end to end, digitally enabled process wherein an eligible vendor can request for a Letter of Recommendation from the Urban Local Body and on receipt of the same he can apply for loan under PM SVANidhi.

The vendors possessing Letter of Recommendation shall be issued Certificate of Vending or ID Card within a period of 30 days. This provision will help in extending reach of the Scheme to the people who were hitherto unable to avail these benefits.

PM SVANidhi was launched by the Ministry of Housing and Urban Affairs, on June 01, 2020, for providing affordable Working Capital loan to street vendors to resume their livelihoods that have been adversely affected due to Covid-19 lockdown. Hence only statement 1 is correct.

QUESTION: Consider the following statements:

1. According to a recent study by TRAFFIC on the seizure and mortality of ‘common leopards’ the highest numbers of poaching incidents were reported from the States of Kerala and Odisha.

2. TRAFFIC is a leading wildlife trade monitoring network across the world.

Which of the statements given above is/are correct?

A. 1 only

B. 2 only

C. Both 1 and 2

D. Neither 1 nor 2

Answer: B

A recent study by TRAFFIC India on the seizure and mortality of ‘common leopards’ (Panthera pardus fusca) revealed that of the total of 747 leopard deaths between 2015-2019 in India, 596 were linked to illegal wildlife trade and activities related to poaching.

TRAFFIC is a leading wildlife trade monitoring network across the world.

The paper titled ‘‘SPOTTED’ in Illegal Wildlife Trade: A Peek into Ongoing Poaching and Illegal Trade of Leopards in India’ also said that the highest numbers of poaching incidents were reported from the States of and Maharashtra.

During the period 2015 to 2019, there were more than 140 cases of seizures of leopard body parts in Uttarakhand, and about 19 incidents where the deaths of these cats could be directly linked to poaching.

Among all the derivatives found in illegal wildlife trade, skin remained the most in-demand product, accounting for 69% of all seizures, while derivatives like claws, teeth and bones were also traded.

Hence only statement 2 is correct.

QUESTION: With reference to the All India Powerloom Board, consider the following statements:

1. The All India Powerloom Board was first constituted as an advisory board in 1981.

2. It was headed by the Union minister of Finance.

Which of the statements given above is/are correct?

A. 1 only

B. 2 only C. Both 1 and 2

D. Neither 1 nor 2

Answer: A

After the all India handicrafts and handloom boards, the ministry of textiles has disbanded another advisory body — the All India Powerloom Board. The All India Powerloom Board was first constituted as an advisory board in 1981. It was reconstituted for a period of two years in 2013. It had representatives of central and state governments, Powerloom Federation and was headed by the Union minister of textiles. The decision to disband the boards follows an assessment that they failed to impact policy-making and became vehicles of “political patronage”. Ministry of Textiles has also notified a change in the status of all eight Textiles Research Associations (TRAs). They now cease to be “affiliated bodies” of the ministry. The TRAs will now be “approved bodies” for conducting testing, research and developmental activities related to the textiles sector. Hence only statement 1 is correct.

QUESTION: With reference to eSanjeevani, consider the following statements:

1. The eSanjeevani platform has enabled two types of telemedicine services viz. Doctor-to-Doctor (eSanjeevani) and Patient-to-Doctor (eSanjeevani OPD) Tele-consultations.

2. The former is being implemented under the Ayushman Bharat Health and Wellness Centre (AB- HWCs) programme.

Which of the statements given above is/are correct?

A. 1 only

B. 2 only

C. Both 1 and 2

D. Neither 1 nor 2

Answer: C

Union Health Minister presided over a review meeting with States/UTs on the “eSanjeevani” and “eSanjeevaniOPD” platforms. In a short span of time since November 2019, tele-consultation by eSanjeevani and eSanjeevaniOPD have been implemented by 23 States (which covers 75% of the population) and other States are in the process of rolling it out. eSanjeevani, the Tele-Medicine platform rolled out by Health Ministry, has completed more than 1,50,000 tele-consultations enabling patient to doctor consultations from the confines of their home, as well as doctor to doctor consultations. This eSanjeevani platform has enabled two types of telemedicine services viz. Doctor- to-Doctor (eSanjeevani) and Patient-to-Doctor (eSanjeevani OPD) Tele-consultations. The former is being implemented under the Ayushman Bharat Health and Wellness Centre (AB-HWCs) programme. Hence only statement 1 is correct. QUESTION: With reference to COVAX platform, consider the following statements:

1. It is an international alliance by WHO aimed at accelerating the development and manufacture of novel Coronavirus vaccines, and ensuring equitable access to all.

2. The facility aims to procure at least two billion doses of a novel Coronavirus vaccine by the end of next year, for deployment and distribution mainly in the low and middle income countries.

Which of the statements given above is/are correct?

A. 1 only

B. 2 only

C. Both 1 and 2

D. Neither 1 nor 2

Answer: C

The World Health Organisation (WHO) has renewed its invite to countries to join its COVAX facility, an international alliance aimed at accelerating the development and manufacture of novel Coronavirus vaccines, and ensuring equitable access to all. The COVAX facility, launched in April 2020, is stitching together agreements with developers and manufacturers of vaccine candidates it thinks are most likely to succeed. The facility is trying to raise funds to support the rapid development of promising candidates, and the quick expansion of manufacturing capacities, so that the vaccines once approved, could be produced on mass scale. The facility aims to procure at least two billion doses of a novel Coronavirus vaccine by the end of next year, for deployment and distribution mainly in the low and middle income countries. The countries who join the initiative are also assured supply of vaccines whenever they become successful. The countries will get assured supplies to protect at least 20 per cent of their populations. Hence both statements are correct.

QUESTION: With reference to the “Transparent Taxation - Honouring the Honest”, consider the following statements:

1. The platform has major reforms like Faceless Assessment, Faceless Appeal and Taxpayers Charter.

2. Faceless Assessment and Taxpayers Charter came into force with immediate effect while the facility of faceless appeal will be available for citizens across the country from 25th September.

Which of the statements given above is/are correct?

A. 1 only

B. 2 only

C. Both 1 and 2

D. Neither 1 nor 2

Answer: C

Prime Minister Modi launched a platform for “Transparent Taxation - Honouring the Honest”. The platform has major reforms like Faceless Assessment, Faceless Appeal and Taxpayers Charter. Faceless Assessment and Taxpayers Charter came into force with immediate effect while the facility of faceless appeal will be available for citizens across the country from 25th September i.e. Deen Dayal Upadhyay's birth anniversary. The new platform apart from being faceless is also aimed at boosting the confidence of the taxpayer and making him/her fearless. The launch of Taxpayers Charter is a significant step where the taxpayer is now assured of fair, courteous and rational behavior. Hence both statements are correct.

QUESTION: With reference to the "Sarabhai" Crater, consider the following statements:

1. It is located on the landing site of Chandrayaan 2’s Vikram lander on the Moon.

2. "Sarabhai" Crater is named after Dr Vikram Sarabhai, the Father of India's Space Programme.

Which of the statements given above is/are correct?

A. 1 only

B. 2 only

C. Both 1 and 2

D. Neither 1 nor 2

Answer: B

ISRO paid tribute to Dr Vikram Sarabhai, the Father of India's Space Programme, in a special way by announcing that Chandrayaan 2 Orbiter has captured the Moon images of “Sarabhai” Crater. "Sarabhai" Crater is named after Dr Vikram Sarabhai. To the 300km east of this crater lie the landing sites of American Apollo 17 and Soviet-launched Luna 21 missions. The Crater has a depth of around 1.7 Kms taken from its raised rim and the slope of Crater walls is in between 25 to 35 degree. These findings will help the Space Scientists to understand further the process on the lunar region filled with lava. Hence only statement 2 is correct.

QUESTION: Consider the following statements regarding 6th Round Table of ASEAN-India Network of Think Tanks (AINTT):

1. The theme of the two day Round Table was 'ASEAN-India: Strengthening Partnership in the Post COVID Era'.

2. It was established at the 7th ASEAN-India Summit in Thailand in 2009 to provide policy inputs to Governments on future direction of cooperation.

Select the correct codes given below: A. 1 only

B. 2 only

C. Both 1 and 2

D. Neither 1 nor 2

Answer: C

The Ministry of External Affairs in partnership with the Foreign Ministry of Thailand has organized the 6th Round Table of ASEAN-India Network of Think Tanks (AINTT).

The theme of the two day Round Table was 'ASEAN-India: Strengthening Partnership in the Post COVID Era'.

More than 400 people from academia, industry, government and diplomatic community attended the Round Table.

The ASEAN-India Round Table comprising think tanks, policy makers, scholars, media and business representatives was established at the 7th ASEAN-India Summit in Thailand in 2009 to provide policy inputs to Governments on future direction of cooperation.

Hence both statements are correct.

QUESTION: The objectives of Prime Minister’s Research Fellows Scheme:

(a) It aims to attract meritorious students into research

(b) It aims to attract all the students in the country to public service

(c) It aims to deploy students in district administration as volunteers

(d) It aims to attract students from rural background into research

Ans: (a) It aims to attract meritorious students into research

In order to attract meritorious students into research, Government of India, in 2018 launched Prime Minister’s Research Fellows (PMRF) Scheme, which offers direct admission to such students in the Ph.D programmes in IITs, IISERs and IISc.

It aims to attract the talent pool of the country to doctoral (Ph.D.) programs of Indian Institutes of Technology (IITs) and Indian Institute of Science (IISc) for carrying out research in cutting edge science and technology domains, with focus on national priorities.

Union HRD Minister announces modifications in PMRF Scheme to boost research in the country. The modifications will enable more students to avail of the benefit under PMRF scheme.

QUESTION: Consider the following statements regarding Vizag Gas Leak and Strict liability principle:

1. The National Green Tribunal Act of 2010 incorporates the strict liability principle.

2. Under it, a party/company is not liable and need not pay compensation if a hazardous substance escapes its premises by accident or by an ‘act of God’ Select the correct codes given below:

(a) 1 only

(b) 2 only

(c) Both 1 and 2

(d) Neither 1 nor 2

Ans: (a) 1 only

Strict Liability Principle:

Under it, a party/company is not liable and need not pay compensation if a hazardous substance escapes its premises by accident or by an ‘act of God’ (Force Majeure) among other circumstances.

Absolute Liability Principle:

Under it, a party/company in a hazardous industry cannot claim any exemption. It has to mandatorily pay compensation, whether or not the disaster was caused by its negligence.

The National Green Tribunal Act of 2010 incorporates the absolute liability principle.

Recently, the National Green Tribunal (NGT) found LG Polymers prima facie liable under the strict liability principle for the Vizag gas leak.

QUESTION: Consider the following statements regarding Global Nutrition Report:

1. India is among 88 countries that are likely to miss global nutrition targets by 2025.

2. It Was conceived following the first Nutrition for Growth Initiative Summit (N4G) in 2013.

Select the correct codes given below:

(a) 1 only

(b) 2 only

(c) Both 1 and 2

(d) Neither 1 nor 2

Ans: (c) Both 1 and 2

The Global Nutrition Report was conceived following the first Nutrition for Growth Initiative Summit (N4G) in 2013 as a mechanism for tracking the commitments made by 100 stakeholders spanning governments, aid donors, civil society, the UN and businesses.

India is among 88 countries that are likely to miss global nutrition targets by 2025. India is also the country with the highest rates of domestic inequalities in malnutrition.

India is identified as among the three worst countries, along with Nigeria and Indonesia, for steep within-country disparities on stunting, where the levels varied four-fold across communities. Global Nutrition Report 2020 has been released.

QUESTION: Consider the following statements regarding Regional Comprehensive Economic Partnership (RCEP):

1. If India joins RCEP it will result in a steep appreciation of rupee.

2. It is a non-free trade agreement originally devised to consist of 16 countries across the Asia-Pacific region.

Select the incorrect codes given below:

(a) 1 only

(b) 2 only

(c) Both 1 and 2

(d) Neither 1 nor 2

Ans: (c) Both 1 and 2

The Regional Comprehensive Economic Partnership is a free trade agreement originally devised to consist of 16 countries across the Asia-Pacific region.

The pact looks to drop tariffs and duties between the members so that goods and services can flow freely between them.

May 15 was the deadline for a response to a fresh proposal of India rejoining negotiations on the ASEAN-led trade Regional Comprehensive Economic Partnership (RCEP).

QUESTION: Consider the following statements regarding Pradhan Mantri Matsya Sampada Yojana (PMMSY):

1. The Scheme will be implemented during a period of 5 years from FY 2020-21 to FY 2024-25.

2. The nodal ministry of this scheme is the Ministry of Shipping.

3. It is a central sector scheme.

Select the correct codes given below:

(a) 1 only

(b) 2 and 3 only

(c) 1 and 3 only

(d) 1 and 2 only

Ans: (a) 1 only

PMMSY is a scheme to bring about Blue Revolution through sustainable and responsible development of fisheries sector in India under two components namely, Central Sector Scheme (CS) and Centrally Sponsored Scheme (CSS). The Scheme will be implemented during a period of 5 years from FY 2020-21 to FY 2024-25.

Nodal Ministry: Ministry of Fisheries, Animal Husbandry and Dairying.

The Union Cabinet has given its approval for implementation of the Pradhan Mantri Matsya Sampada Yojana (PMMSY).

QUESTION: Scientists in the Netherlands have discovered a potential new organ named “tubarial salivary glands”. In this context, consider the following statements: 1. This gland lubricates and moistens the upper throat behind the nose and mouth.

2. Until now, there were only three known large salivary glands in humans.

Which of the above statements is/are correct?

(a) 1 only

(b) 2only

(c) Both 1 and 2

(d) Neither 1 nor 2

Answer: (c)

Both statements are correct

Tubarial Salivary Glands Researchers at the Netherlands Cancer Institute have identifi ed a set of salivary glands deep in the upper part of the throat and have named them “tubarial salivary glands”.

The discovery may be important for cancer treatment. So far, this nasopharynx region

— behind the nose — was not thought to host anything but microscopic, diffuse, salivary glands.

The newly discovered glands are about 1.5 inches (3.9 centimeters) in length on average and are located over a piece of cartilage called the torus tubarius.

The glands probably lubricate and moisten the upper throat behind the nose and mouth.

Until now, there were three known large salivary glands in humans: one under the tongue, one under the jaw and one at the back of the jaw, behind the cheek.

The new organ was discovered while scientists were studying prostate cancer cells using PSMA PET-CT technology — a combination of CT scans and positron emission tomography (PET) — which is good in detecting salivary gland tissues.

In this technique, a radioactive “tracer” is injected into the patient that binds to the protein PSMA, which is elevated in prostate cancer cells.

QUESTION: Recently, a giant cat geoglyph was discovered on a hill at the famous Nazca Lines site in Peru. In this context, consider the following statements:

1. Geoglyphs are the large designs made on the ground by creators using elements of the landscape such as stones, gravel, dirt or lumber.

2. Nazca Lines are recognized as a World Heritage Site by UNESCO.

Which of the above statements is/are correct?

(a) 1 only

(b) 2 only

(c) Both 1 and 2

(d) Neither 1 nor 2

Answer: (c)

Both statements are correct

Nazca Lines

Recently, a giant cat geoglyph was discovered on a hill at the famous Nazca Lines site in

Peru.

Nazca Lines are a group of geoglyphs known for the depictions of larger-than-life animals, plants and imaginary beings.

Geoglyphs are the large designs made on the ground by creators using elements of the landscape such as stones, gravel, dirt or lumber.

These are believed to be the greatest known archaeological enigma, owing to their size, continuity, nature and quality.

The Lines were first discovered in 1927, and were declared a World Heritage Site by

UNESCO in 1994.

QUESTION: With reference to Ayushman Sahakar initiative, consider the following statements:

1. It covers establishment, modernization, expansion, repairs, renovation of hospital and healthcare and education infrastructure.

2. It is formulated by the National Cooperative Development Corporation (NCDC).

3. NCDC is the apex autonomous development finance institution under the Ministry of Finance.

Which of the above statements are correct? (a) 1 and 2 only

(b) 1 and 3 only

(c) 2 and 3 only

(d) 1, 2 and 3

Answer: (a)

Statement 3 is incorrect: NCDC was set up under an Act of Parliament in 1963 for promotion and development of cooperatives.

It functions under the Ministry of Agriculture and Farmers Welfare.

Ayushman Sahakar:

Union Minister for Agriculture launched AYUSHMAN SAHAKAR, a unique scheme to assist cooperatives to play an important role in creation of healthcare infrastructure in the country.

The scheme is formulated by the National Cooperative Development Corporation (NCDC), the apex autonomous development finance institution under the Ministry of Agriculture and Farmers Welfare.

NCDC would extend term loans to prospective cooperatives to the tune of Rs.10,000 Crore in the coming years.

There are about 52 hospitals across the country run by cooperatives. The NCDC fund would give a boost to provision of healthcare services by cooperatives.

Ayushman Sahakar specifically covers establishment, modernization, expansion, repairs, renovation of hospital and healthcare and education infrastructure.

Any Cooperative Society with suitable provision in its byelaws to undertake healthcare related activities would be able to access the NCDC fund. NCDC assistance will flow either through the State Governments/ UT Administrations or directly to the eligible cooperatives.

QUESTION: Consider the following statements regarding ‘Life in Miniature’ Project:

1. It will allow online viewing of various hundred small-scale paintings from the National Museum, New Delhi on Google Arts & Culture by people all over the world.

2. It spotlights the role of technology in the conservation of India’s heritage.

3. National Museum is the premiere cultural institution of the India under Ministry of Culture.

Which of the above statements is/are correct?

(a) 1 only

(b) 1 and 2 only (c) 2 and 3 only

(d) 1, 2 and 3

Answer: (d)

All statements are correct

Life in Miniature Project:

Union Minister for Culture and Tourism virtually launched “Life in Miniature” project, a collaboration between the National Museum, New Delhi, Ministry of Culture, and Google Arts& Culture.

Several hundred miniature paintings from the National Museum, New Delhi can be viewed online on Google Arts & Culture by people around the world in a new project titled “Life in Miniature.”

The project uses technologies like machine learning, augmented reality and digitization with high-definition robotic cameras, to showcase these special works of art in a magical new way.

On the Google Arts & Culture app, online viewers can experience the first Augmented Reality-powered art gallery designed with traditional Indian architecture, and explore a life-size virtual space where you can walk up to a selection of miniature paintings.

Google Arts & Culture is an immersive way to explore art, history and the wonders of the world. The Google Arts & Culture app is free and available online for iOS and

Android.

QUESTION: Government has recently banned mobile applications like PUBG. In this context, consider the following statements:

1. Section 69A of Information Technology Act provides the government with emergency powers to block any content.

2. As per Section 69A of IT Act, prior notice must be served to content providers before ordering a ban.

3. Blocking results in the removal of content by an online platform.

Which of the above statements are incorrect ?

(a) 1 and 2 only

(b) 1 and 3 only (c) 2 and 3 only

(d) 1, 2 and 3

Answer: (c)

Statement 2 is incorrect: Sec 69A provides government with emergency powers to hold enquiry after the ban. However the Blocking Rules, 2009 specifically provide for a defined process of notice, hearing and a reasoned order. These processes emerge from the Shreya Singhal judgment (2008) and apply to all grounds for blocking, including those premised on national security.

Statement 3 is incorrect: A block disables access for a user in India through an internet service provider such as Airtel whereas a takedown results in the removal of content by an online platform such as Facebook.

Section 69A of the Information Technology Act:

The government banned 118 more mobile applications, including the popular gaming app PUBG, that it said are prejudicial to the sovereignty and integrity of India.

The decision to ban such Apps was taken by the Ministry of Electronics and Information Technology which invoked it’s power under section 69A of the Information Technology Act read with the relevant provisions of the Information Technology (Procedure and Safeguards for Blocking of Access of Information by Public) Rules 2009.

Under this act, the government has emergency powers to block any content where it is satisfied that it is against the country’s sovereignty, integrity, and defense, and in such cases the law does not require it serving any prior notice to the concerned content providers before ordering a ban.

It also provides government with emergency powers to hold enquiry after the ban

QUESTION: Which of the following statements regarding Aatmanirbhar Bharat ARISE-ANIC Initiative is/are correct?

1. It is a national initiative to promote research & innovation and increase competitiveness of Indian startups and MSMEs.

2. It will provide funding support to deserving applied research–based innovations.

3. It will be driven by Indian Space Research Organization (ISRO).

Select the correct answer using the codes given below:

(a) 1 only

(b) 1 and 2 only

(c) 3 only (d) 1, 2 and 3

Answer: (d)

All statements are correct

Aatmanirbhar Bharat ARISE-ANIC Initiative:

Atal Innovation Mission (AIM), NITI Aayog, launched its programme, the Aatmanirbhar Bharat ARISE- Atal New India Challenges, to spur applied research and innovation in Indian MSMEs and startups.

The programme will be driven by Indian Space Research Organization (ISRO), four ministries— Ministry of Defence; Ministry of Food Processing Industries; Ministry of Health and Family Welfare; and Ministry of Housing and Urban Affairs—and associated industries to facilitate innovative solutions to sectoral problems.

The Aatmanirbhar Bharat ARISE–ANIC programme will support deserving applied research–based innovations by providing funding support of up to Rs 50 lakh for speedy development of the proposed technology solution and/or product.

The objective of Aatmanirbhar Bharat ARISE-ANIC program is to proactively collaborate with esteemed Ministries and the associated industries to catalyze research, innovation and facilitate innovative solutions to sectoral problems.

The objective is also to provide a steady stream of innovative products & solutions where the Central Government Ministries / Departments will become the potential first buyers.

QUESTION: Consider the following statements regarding Climate Smart Cities Assessment Framework (CSCAF) 2.0:

1. It aims to support cities to develop unified vision of streets in consultation with stakeholders.

2. It will be implemented by National Institute of Urban Affairs (NIUA).

3. It has been launched under the .

Which of the above statements is/are correct?

(a) 1 only

(b) 1 and 2 only

(c) 2 and 3 only

(d) 1, 2 and 3

Answer: (c)

Statement 1 is incorrect:Streets for people challenge, launched alongside Climate Smart Cities Assessment Framework (CSCAF) 2.0 aims to support cities to develop unified vision of streets in consultation with stakeholders and inspire cities to create walking-friendly/ vibrant streets through quick, innovative/ low-cost measures. All participating cities to be encouraged to use ‘test-learn- scale’ approach to initiate both, flagship and neighborhood walking interventions

Climate Smart Cities Assessment Framework (CSCAF) 2.0 :

The Ministry of Housing and Urban Affairs has launched the Climate Smart Cities Assessment Framework (CSCAF) 2.0 under the Smart Cities Mission.

The objective of CSCAF is to provide a clear roadmap for cities towards combating

Climate Change while planning and implementing their actions, including investments.

CSCAF initiative intends to inculcate a climate-sensitive approach to urban planning and development in India.

The framework has 28 indicators across five categories namely; (i) Energy and Green

Buildings, (ii) Urban Planning, Green Cover & Biodiversity, (iii) Mobility and Air Quality, (iv) Water Management and (v) Waste Management.

The Climate Centre for Cities under National Institute of Urban Affairs (NIUA) is supporting MoHUA in implementation of CSCAF

QUESTION: With reference to Scramjet vehicle, consider the following statements:

1. These are a variant of a category of jet engines called the air breathing engines.

2. They can help in achieving hypersonic speeds.

3. They offer the advantage of high thrust-to-weight ratio.

Which of the above statements is/are correct?

(a) 1 and 2 only

(b) 2 only

(c) 1 and 3 only

(d) 3 only

Answer: (a)

Statement 3 is incorrect: High thrust-to-weight ratio is its disadvantage.

Scramjet Vehicle :

The Defence Research and Development Organization (DRDO) on Monday successfully f ight tested the Hypersonic Technology Demonstrator Vehicle (HSTDV)

– an unmanned scramjet vehicle with a capability to travel at six times the speed of sound.

The indigenous development of the technology will also boost the development of the systems built with hypersonic vehicles at its core, including both offensive and defensive hypersonic cruise missile systems and also in the space sector. The scramjets are a variant of a category of jet engines called the air breathing engines. The ability of engines to handle air flows of speeds in multiples of speed of sound, gives it a capability of operating at those speeds.

Hypersonic speeds are those which are five times or more than the speed of sound. The unit tested by the DRDO can achieve upto six times the speed of sound or Mach 6, which is well over 7000 kilometers per hour or around two kilometers per second.

While the technology helps achieve hypersonic speeds, it comes with its set of disadvantages, and the obvious one being its very high cost and high thrust-to-weight ratio.

QUESTION: Recently an international team of astronomers has announced the finding of phosphine gas in the atmosphere of Venus. In this context, consider the following statements:

1. Phosphine is a colorless gas produced by some species of aerobic bacteria.

2. Geochemical processes like volcanic or meteorite activity is known to produce it.

Which of the above statements is/are correct?

(a) 1 only

(b) 2 only

(c) Both 1 and 2

(d) Neither 1 nor 2

Answer: (b)

Statement 1 is incorrect: Phosphine, a colorless, smelly gas, is known to be made only by some species of bacteria that survive in the absence of oxygen.

Phosphine Gas An international team of astronomers has announced the finding of phosphine gas in the atmosphere of Venus, triggering excitement about the possibility of presence of life forms on that planet.

Apart from being produced in industrial processes, phosphine, a colorless, smelly gas, is known to be made only by some species of bacteria that survive in the absence of oxygen. The presence of phosphine in the atmosphere of Venus is something that was not expected and is “unexplained”. Any presence of phosphorus in that atmosphere was expected to be in oxidized forms.

In a paper published in Nature Astronomy, a team of scientists have reported traces of phosphine in a concentration of approximately 20 parts per billion. Scientists have been careful to emphasize, that as of now, this is no confirmation of the presence of life on Venus.

QUESTION: Consider the following statements:

1. The Aircraft Act of 1934 makes provisions for the control of the manufacture, use and sale of aircrafts. 2. The Aircraft (Amendment) Bill 2020 seeks to provide statutory status to the Directorate General of Civil Aviation (DGCA).

3. India is a signatory to the Chicago Convention on international civil aviation.

Which of the above statements are correct?

(a) 1 and 2 only

(b) 2 and 3 only

(c) 1 and 3 only

(d) 1, 2 and 3

Answer: (d)

All statements are correct

The Aircraft (Amendment) Bill 2020 Rajya Sabha recently passed The Aircraft (Amendment) Bill 2020, which seeks to provide statutory status to the Directorate General of Civil Aviation (DGCA), the Bureau of Civil Aviation Security (BCAS), and the Aircraft Accidents Investigation Bureau (AAIB).

The latest amendments to the Aircraft Act of 1934 seek to expand the role of the two regulators — DGCA and BCAS — and of the AAIB.

The DGCA, which is the aviation safety regulator, will be empowered to impose penalties for certain violations in addition to increasing the maximum penalty limit to Rs 1 crore from the existing Rs 10 lakh.

However, the amendments will also now allow the Ministry of Civil Aviation to review any order passed by the Director General of Civil Aviation and the Director General of Civil Aviation Security, and also direct them to rescind or modify such order.

The Aircraft Act of 1934 was enacted to make provisions for the control of the manufacture, possession, use, operation, sale, import and export of aircraft.

It makes provisions for securing the safety of aircraft operations in India, and for carrying out civil aviation operations as per internationally accepted standards, procedures and practices as laid down by the International Civil Aviation Organisation (ICAO).

The ICAO, under its Universal Safety Oversight Audit Programme and the Universal Security Audit Programme, regularly conducts safety and security audits of all countries which are signatory to the Chicago Convention to ensure they are carrying out their safety and security oversight functions. India, as a signatory, is also subjected to periodic audits by ICAO and the FAA.

According to government sources, the audits conducted by the ICAO in 2012 and 2015 indicated a need to amend the Aircraft Act to give proper recognition to the regulators under the Act, to enhance the maximum quantum of fines and to empower the departmental officers to impose financial penalties on individuals or organizations involved in violations of the legal provisions and to include certain areas of air navigation services for rulemaking purposes under Section 5 of the Act.

QUESTION: Consider the following statements regarding FAME India Scheme:

1. It aims to promote adoption of electronic as well hybrid vehicles in India.

2. It is a scheme of Ministry of Electronics and Communication Technology (MeitY).

3. Privately owned vehicles are not covered under this scheme.

Which of the above statements is/are correct?

(a) 1 only

(b) 1 and 3 only

(c) 2 and 3 only

(d) 1 and 2 only

Answer: (a)

Statement 2 is incorrect: It is a scheme of the Ministry of Heavy Industries and Public Enterprises.

Statement 3 is incorrect: Vehicles, fitted with only advanced chemistry battery, meeting with minimum Technical Criteria and registered as “Motor Vehicle” as per CMVR shall be eligible for incentive under the scheme. With greater emphasis on providing affordable & environment friendly public transportation options for the masses, scheme will be applicable mainly to vehicles used for public transport or those registered for commercial purposes in e-3W, e-4W and e-bus segments. However, privately owned registered e-2Ws are also covered under the scheme as a mass segment.

FAME India Scheme

In a big push towards electric mobility the Government has sanctioned 670 Electric buses in the states of Maharashtra, Goa, Gujarat and Chandigarh and 241 Charging Stations in Madhya Pradesh, Tamil Nadu, Kerala, Gujarat and Port Blair under Phase- II of FAME India Scheme.

The Department of Heavy Industries under the Ministry of Heavy Industries and Public Enterprises, is administering the Faster Adoption and Manufacturing of (Hybrid &) Electric Vehicles in India (FAME India) Scheme since April, 2015 to promote adoption of electric and hybrid vehicles (xEVs) in India.

At present, Phase-II of FAME India Scheme is being implemented for a period of 3 years with effect from April 1, 2019 with a total budgetary support of Rs 10,000 crore.

This phase focuses on supporting electrification of public & shared transportation and aims to support, through subsidies, approximately 7,000 e-buses, 5 lakh e-3 wheelers, 55,000 e-4 wheeler passenger cars and 10 lakh e-2 wheelers.

QUESTION: KRITAGYA, seen in news recently,is a:

(a) Environmental single window hub for Environment, Forest, Wildlife and CRZ clearances. (b) People centric fuel conservation campaign.

(c) Ag-Tech hackathon to promote innovation in farm mechanization.

(d) Web-portal to ensure coal supplies to power plants

Answer: (c)

KRITAGYA

In order to promote potential technology solutions for enhancing farm mechanization with special emphasis on women friendly equipment’s, a hackathon named “KRITAGYA” has been planned by the Indian Council of Agricultural Research (ICAR) under National Agricultural Higher Education Project (NAHEP).

Students, faculties and innovators/entrepreneurs from any university / technical institution across the country can apply and participate in the event in the form of a group.

In one group maximum 4 participants can compete, with not more than one faculty and / or more than one innovator or entrepreneur. Participating students can collaborate with local start-ups, students from technology institutes, and can win Rs. 5 lakhs, Rs 3 lakhs and Rs. 1 lakh as first, second and third prize.

QUESTION: With reference to Regional Raw Drug Repository (RRDR), consider the following statements: 1. RRDR’s are setup by National Medicinal Plants Board (NMPB) under Ministry of AYUSH.

2. RRDR would act as an accredited reference library for authentication of raw drugs.

3. NMPB is the nodal agency for management of plant genetic resources of all agri-horticultural crops.

Which of the above statements is/are incorrect ?

(a) 1 and 2 only

(b) 1 and 3only

(c) 2 only

(d) 3 only

Answer: (d)

Statement 3 is incorrect: National Bureau of Plant Genetic Resources (NBPGR) is the nodal agency for management of plant genetic resources of all agri-horticultural crops.NMBP focuses on in-situ & ex- situ conservation and augmenting local medicinal plants and aromatic species of medical significance. The NMPB also promote research & development, capacity building through trainings, raising awareness through promotional activities like creation of Home/School herbal gardens

Regional Raw Drug Repository (RRDR) The Minister for AYUSH recently inaugurated the Regional Raw Drug Repository (RRDR) today at All India Institute of Ayurveda, New Delhi. This RRDR is the second in the series of repositories proposed by National Medicinal Plants Board (NMPB), Ministry of AYUSH and will be dedicated to the Trans-Ganga Plain Region.

In order to document the diversity in the genetics and chemistry of medicinal plants, eight RRDR and one NRDR have been proposed by Ministry of AYUSH. Out of them, three Regional Raw Drug Repositories are ready.

RRDR for Trans- Ganga Plain Region covers four states -Haryana, Chandigarh, Delhi and Punjab. This RRDR would play a stellar role in collection, documentation, and authentication of raw drugs collected largely from the respective agro-climatic region.

The key objectives of establishing these repositories are to: Act as a collection center of raw drugs available and used in each region.

Act as an accredited reference library for authentication of raw drugs.

Establish standard protocols and keys for authentication of raw drug used in the herbal industries.

Act as an educational Centre for disseminating general awareness about the usefulness of raw drugs.

QUESTION: With reference to World Food Programme, consider the following statements:

1. It is world’s largest humanitarian organization focused on hunger and food security.

2. Majority of its work is confined to conflict- affected countries.

3. It is headquartered in Geneva.

Which of the above statements is/are incorrect ?

(a) 1 and 2 only

(b) 2 only

(c) 2 and 3 only

(d) 3 only

Answer: (d)

Statement 3 is incorrect: It is headquartered in Rome.

World Food Programme

The Norwegian Nobel Committee has decided to award the Nobel Peace Prize for 2020 to the World Food Programme (WFP) for its efforts to combat hunger.

The World Food Programme (WFP) is the food-assistance branch of the United Nations and the world’s largest humanitarian organization focused on hunger and food security.

Founded in 1961, it is headquartered in Rome. The World Food Programme was an active participant in the diplomatic process that culminated in May 2018 in the UN Security Council’s unanimous adoption of Resolution 2417, which for the first time explicitly addressed the link between conflict and hunger.

WFP’s efforts focus on emergency assistance, relief and rehabilitation, development aid and special operations. Two-thirds of its work is in conflict-affected countries where people are three times more likely to be undernourished than those living in countries without conflict.

QUESTION: The Global Social Mobility Index is prepared by a) United Nations Population Fund b) World Bank c) World Economic Forum d) Amnesty International

Ans) c

The Global Social Mobility Index is an index prepared by the World Economic Forum in the Global Social Mobility report. The Index measures the inter-generational social mobility in different countries in relation to socioeconomic outcomes. Recently, WEF has come out with its first-ever Global Social Mobility Report, 2020.

The inaugural ranking ranked 82 countries. India ranks 76th out of 82 economies. It ranks 41st in lifelong learning and 53rd in working conditions.

The Index assesses the 82 economies on “10 pillars” spread across the following five key dimensions of social mobility:

1) Health;

2) Education (access, quality and equity, lifelong learning);

3) Technology;

4) Work (opportunities, wages, conditions);

5) Protection and Institutions (social protection and inclusive institutions).

QUESTION: The NISHTHA programme was launched by the Government to a) Provide handholding support to differently abled for availing skill training. b) Protect the border areas by enhancing surveillance and intelligence capabilities. c) Provide technological support to the micro, small and medium enterprises. d) Improve quality of school education through integrated teacher training.

Ans) d NISHTHA stands for National Initiative for School Heads and Teachers Holistic Advancement.

The NISHTHA programme was launched by the Department of School Education and Literacy, Ministry of Human Resource Development in 2019. NISHTHA is a capacity building programme for "Improving Quality of School Education through Integrated Teacher Training". It aims to build competencies among all the teachers and school principals at the elementary stage. NISHTHA is the world's largest teachers' training programme of its kind. The basic objective of this massive training programme is to motivate and equip teachers to encourage and foster critical thinking in students. The initiative is first of its kind wherein standardized training modules are developed at national level for all States and UTs.

QUESTION: With regard to the National Data and Analytics Platform, consider the following statements:

1. It was released by the Ministry of Electronics and Information Technology.

2. It aims to democratize access to publicly available government data.

Which of the statements given above is/are correct? a) 1 only b) 2 only c) Both 1 and 2 d) Neither 1 nor 2

Ans) b

Statement 1 is incorrect. The NITI Aayog has released the National Data and Analytics Platform (NDAP).

National Data and Analytics Platform will follow a user-centric approach and will enable data access in a simple and intuitive portal tailored to the needs of a variety of stakeholders.

Statement 2 is correct. The National Data and Analytics Platform (NDAP) aims to democratize access to publicly available government data. It will host the latest datasets from various government websites, present them coherently, and provide tools for analytics and visualization.

QUESTION: With regard to NavIC, consider the following statements: 1. It provides accurate position information service to users only in India.

2. It is an independent regional navigation satellite system developed by ISRO.

Which of the statements given above is/are correct ? a) 1 only b) 2 only c) Both 1 and 2 d) Neither 1 nor 2

Ans) b

Statement 1 is incorrect. Indian Regional Navigation Satellite System (IRNSS), NavIC provides accurate position information service to users in India as well as the region extending up to 1500 km from its boundary which is its primary service area. The space segment consists of the IRNSS constellation of eight satellites.

Statement 2 is correct. Indian Regional Navigation Satellite System (IRNSS), NavIC is an independent regional navigation satellite system developed by the Indian Space Research Organization (ISRO).

Indian Regional Navigation Satellite System (IRNSS), NavIC Applications:

1) Terrestrial, Aerial and Marine Navigation

2) Disaster Management

3) Vehicle tracking and fleet management

4) Integration with mobile phones

5) Precise Timing

6) Mapping and Geodetic data capture

7) Terrestrial navigation aid for hikers and travelers

8) Visual and voice navigation for drivers.

QUESTION: The Reskilling Revolution Initiative has been launched by a) World Economic Forum b) World Bank c) European Union d) World Trade Organization

Ans) a

The World Economic Forum has launched the Reskilling Revolution. It is an initiative to provide one billion people with better education, skills and jobs by 2030. The scheme aims to future-proof workers from technological change and help economies by providing new skills for the Fourth Industrial Revolution.

Founding governments include Brazil, France, India, Pakistan, the Russian Federation, UAE and the US. Business partners include PwC, Salesforce, ManpowerGroup, Infosys, LinkedIn, Coursera Inc. and The Adecco Group. The Reskilling Revolution platform has been designed to prepare the global workforce with the skills needed to future-proof their careers against the expected displacement of millions of jobs and skills instability as a result of technological change. It is also designed to provide businesses and economies with the skilled labour needed to fulfil the millions of new roles that will be created by the Fourth Industrial Revolution, shifts in the global economy and industrial transitions towards sustainability.

QUESTION: Consider the following statements regarding Biotechnology Industry Research Assistance Council (BIRAC):

1. It is a not-for-profit Public Sector Enterprise.

2. It was set up by the Department of Biotechnology (DBT), Government of India.

Which of the statements given above is/are correct?

A) 1 only

B) 2 only

C) Both 1 and 2

D) Neither 1 nor 2

Answer : C

Both the statements given above are correct.

Biotechnology Industry Research Assistance Council (BIRAC) is a not-for-profit Section 8, Schedule B, Public Sector Enterprise, set up by Department of Biotechnology (DBT), Government of India as an Interface Agency to strengthen and empower the emerging Biotech enterprise to undertake strategic research and innovation, addressing nationally relevant product development needs.

BIRAC is an industry-academia interface and implements its mandate through a wide range of impact initiatives, be it providing access to risk capital through targeted funding, technology transfer, IP management and handholding schemes that help bring innovation excellence to the biotech firms and make them globally competitive.

QUESTION: Recently, Parliamentary Panel on Law and Justice has submitted its report “Functioning of the Virtual Courts/ Courts Proceedings through Video-Conferencing”. Which of the following is/are its recommendations?

1. Disband the virtual courts in a post-COVID scenario.

2. Transfer of certain categories of cases from regular court establishments to virtual courts.

3. Upgradation of infrastructure for smooth functioning of the virtual courts.

Select the correct answer using the code given below: A. 1 and 2 only

B. 2 and 3 only

C. 1 and 3 only

D. None

Answer : B

Parliamentary Panel on Law and Justice has submitted its report “Functioning of the Virtual Courts/ Courts Proceedings through Video-Conferencing”.

This is the first report to be presented by any parliamentary panel on the impact of the pandemic.

Key recommendations:

Continue virtual courts even in a post-COVID scenario. Hence, statement 1 is incorrect.

Transfer of certain categories of cases, like cases pertaining to traffic challans or other petty offences, from regular court establishments to virtual courts will reduce the pendency of cases. Hence, statement 2 is correct.

Infrastructure needs to be upgraded especially in district courts to implement this. Hence, statement 3 is correct.

Ministry of Law and Justice and Ministry of Electronics and Information Technology should address data privacy and data security concerns while developing a new platform for India’s judicial system.

Virtual Court is a concept aimed at eliminating presence of litigant or lawyer in the court and adjudication of the case online

QUESTION: Section 8 in the Representation of the People Act, 1951, sometimes seen in the news, deals with: A. Model Code of Conduct (MCC) B. Disqualification of a candidate on conviction for certain offences C. Exit Polls D. Provision of the None of the above (NOTA) option. Answer : B Currently, under the Representation of Peoples (RP) Act, lawmakers cannot contest elections only after their conviction in a criminal case. Section 8 of the Representation of the People (RP) Act, 1951 disqualifies a person convicted with a sentence of two years or more from contesting elections. Hence, option (b) is correct.

QUESTION: “Streets for People Challenge” has been recently launched by: A. Ministry of Social Justice and Empowerment B. NITI Aayog C. Red Cross organization D. Ministry of Housing and Urban Affairs Answer : D Housing and Urban Affairs Minister Hardeep Singh Puri recently launched the Climate Smart Cities Assessment Framework, CSCAF 2.0, along with the ‘Streets for People Challenge’ in a virtual event. The objective of CSCAF is to provide a clear roadmap for cities towards combating Climate Change while planning and implementing their actions, including investments. CSCAF initiative intends to inculcate a climate-sensitive approach to urban planning and development in India. The Streets for People Challenge is the response to the need for making the cities more walkable and pedestrian friendly. The Challenge builds on the advisory issued by the Ministry for the holistic planning for pedestrian-friendly market spaces, earlier this year. The Challenge will support cities across the country to develop a unified vision of streets for people in consultation with stakeholders and citizens. Hence, option (d) is correct.

QUESTION: Consider the following statements regarding PM Street Vendor”s Atma Nirbhar Nidhi (PM SVANidhi) 1. It is a special micro-credit facility plan to provide affordable loan of up to ₹10,000 to more than 50 lakh street vendors. 2. NABARD is the technical partner for implementation of this scheme. 3. Technical Partner will manage the credit guarantee to the lending institutions through Credit Guarantee Fund Trust for Micro and Small Enterprises. Which of the statements given above is/are not correct? A. 2 only B. 1 and 3 only C. 1 and 2 only D. None

Answer : A The initiative under PM Street Vendor’s Atma Nirbhar Nidhi or PM SVANidhi Scheme will help these vendors grow their business which was severely hit and are still struggling due to the Covid-19 pandemic. Overview of the scheme: 1. It is a special micro-credit facility plan to provide affordable loan of up to ₹10,000 to more than 50 lakh street vendors, who had their businesses operational on or before 24 March 2020. Hence Statement 1 is correct. 2. The scheme is valid until March 2022. 3. Small Industries Development Bank of India is the technical partner for implementation of this scheme. Hence Statement 2 is incorrect. 4. It will manage the credit guarantee to the lending institutions through Credit Guarantee Fund Trust for Micro and Small Enterprises. Hence Statement 3 is correct. Loans under the scheme: 1. Under the scheme, vendors can avail working capital loan of up to ₹10,000, which is repayable in monthly instalments within one year. 2. On timely/early repayment of the loan, an interest subsidy of 7% per annum will be credited to the bank accounts of beneficiaries through (DBT) on six-months basis. 3. There will be no penalty on early repayment of loan. Eligibility: The scheme is applicable to vendors, hawkers, thelewalas, rehriwalas, theliphadwalas in different areas/contexts who supply goods and services. Street vendors belonging to the surrounding peri-urban/rural areas are also included